You are on page 1of 118

Many electric circuits are complex, but it is an engineers goal to reduce their complexity to

analyze them easily. In the previous chapters, we have mastered the ability to solve networks
containing independent and dependent sources making use of either mesh or nodal analysis. In
this chapter, we will introduce new techniques to strengthen our armoury to solve complicated
networks. Also, these new techniques in many cases do provide insight into the circuits operation
that cannot be obtained from mesh or nodal analysis. Most often, we are interested only in the
detailed performance of an isolated portion of a complex circuit. If we can model the remainder
of the circuit with a simple equivalent network, then our task of analysis gets greatly reduced and
simplified. For example, the function of many circuits is to deliver maximum power to load such
as an audio speaker in a stereo system. Here, we develop the required relationship betweeen a
load resistor and a fixed series resistor which can represent the remaining portion of the circuit.
Two of the theorems that we present in this chapter will permit us to do just that.

3.1

Superposition theorem

The principle of superposition is applicable only for linear systems. The concept of superposition
can be explained mathematically by the following response and excitation principle :
i1
i2

then;

i1

+ i2

!
!
!

v1
v2
v1

+ v2

The quantity to the left of the arrow indicates the excitation and to the right, the system
response. Thus, we can state that a device, if excited by a current i1 will produce a response
v1 . Similarly, an excitation i2 will cause a response v2 . Then if we use an excitation i1 + i2 , we
will find a response v1 + v2 .
The principle of superposition has the ability to reduce a complicated problem to several easier
problems each containing only a single independent source.

160

Network Theory

Superposition theorem states that,


In any linear circuit containing multiple independent sources, the current or voltage at any
point in the network may be calculated as algebraic sum of the individual contributions of each
source acting alone.
When determining the contribution due to a particular independent source, we disable all
the remaining independent sources. That is, all the remaining voltage sources are made zero by
replacing them with short circuits, and all remaining current sources are made zero by replacing
them with open circuits. Also, it is important to note that if a dependent source is present, it must
remain active (unaltered) during the process of superposition.
Action Plan:
(i) In a circuit comprising of many independent sources, only one source is allowed to be active
in the circuit, the rest are deactivated (turned off).
(ii) To deactivate a voltage source, replace it with a short circuit, and to deactivate a current
source, replace it with an open circuit.
(iii) The response obtained by applying each source, one at a time, are then added algebraically
to obtain a solution.
Limitations: Superposition is a fundamental property of linear equations and, therefore, can be
applied to any effect that is linearly related to the cause. That is, we want to point out that,
superposition principle applies only to the current and voltage in a linear circuit but it cannot be
used to determine power because power is a non-linear function.
EXAMPLE

3.1

Find the current in the 6 resistor using the principle of superposition for the circuit of Fig. 3.1.

Figure 3.1
SOLUTION

As a first step, set the current source to zero. That is, the current source appears as an open circuit
as shown in Fig. 3.2.
6
6
= A
i1 =
3+6
9

Circuit Theorems

161

As a next step, set the voltage to zero by replacing it with a short circuit as shown in Fig. 3.3.
2 3
6
i2 =
= A
3+6
9

Figure 3.2

Figure 3.3

The total current i is then the sum of i1 and i2


i

EXAMPLE

= i1 + i2 =

12
A
9

3.2

Find io in the network shown in Fig. 3.4 using superposition.

Figure 3.4
SOLUTION

As a first step, set the current source to zero. That is, the current source appears as an open circuit
as shown in Fig. 3.5.

Figure 3.5

162

Network Theory

6
= 0:3 mA
(8 + 12) 103
As a second step, set the voltage source to zero. This means the voltage source in Fig. 3.4 is
replaced by a short circuit as shown in Figs. 3.6 and 3.6(a). Using current division principle,
0

iA

io

where

R1

iR2
R1

+ R2

jj

= (12 k 12 k) + 12 k
= 6 k + 12 k
= 18 k

and

= 12 k
4 10 3 12 103
iA =
(12 + 18) 103
= 1:6 mA
Again applying the current division principle,
12
iA
00
io =
= 0:8 mA
12 + 12

R2

 


Thus;

io

= io 0 + io 00 =

0:3 + 0:8 = 0:5 mA

Figure 3.6(a)

Figure 3.6

Circuit Theorems

EXAMPLE

163

3.3

Use superposition to find io in the circuit shown in Fig. 3.7.

Figure 3.7

SOLUTION

As a first step, keep only the 12 V source active and rest of the sources are deactivated. That is,
2 mA current source is opened and 6 V voltage source is shorted as shown in Fig. 3.8.

io

12
(2 + 2) 103
= 3 mA
=

Figure 3.8

As a second step, keep only 6 V source active. Deactivate rest of the sources, resulting in a
circuit diagram as shown in Fig. 3.9.

164

Network Theory

Applying KVL clockwise to the upper loop, we get

 10

io

00

 10

io

00

io

00

6=0
6
= 1:5 mA
103

Figure 3.9

As a final step, deactivate all the independent voltage sources and keep only 2 mA current
source active as shown in Fig. 3.10.

Figure 3.10

Current of 2 mA splits equally.


Hence;

io

000

= 1mA

Applying the superposition principle, we find that


io

= io 0 + io 00 + io 000
=3

1:5 + 1

= 2:5 mA

Circuit Theorems

EXAMPLE

165

3.4

Find the current i for the circuit of Fig. 3.11.

Figure 3.11
SOLUTION

We need to find the current i due to the two independent sources.


As a first step in the analysis, we will find the current resulting from the independent voltage
source. The current source is deactivated and we have the circuit as shown as Fig. 3.12.
Applying KVL clockwise around loop shown in Fig. 3.12, we find that

5i1 + 3i1

24 = 0
24
i1 =
= 3A
8

As a second step, we set the voltage source to zero and determine the current
current source. For this condition, refer to Fig. 3.13 for analysis.

Figure 3.12

i2

due to the

Figure 3.13

Applying KCL at node 1, we get


i2

+7=

Noting that

i2

we get,

v1

3i2

v1

2
v1

3
3i2

(3.1)

0
(3.2)

166

Network Theory

Making use of equation (3.2) in equation (3.1) leads to


3i2 3i2
i2 + 7 =
2
7
i2 =
A
4
Thus, the total current

EXAMPLE

= i1 + i2
7
5
=3
A= A
4
4

3.5

For the circuit shown in Fig. 3.14, find the terminal voltage Vab using superposition principle.

Figure 3.14

SOLUTION

As a first step in the analysis, deactivate the independent current source. This results in a circuit diagram as shown in Fig. 3.15.
Applying KVL clockwise gives
4 + 10

0+3
)
)

Vab1

+ Vab1 = 0
4Vab1 = 4
Vab1

= 1V

Next step in the analysis is to deactivate the


independent voltage source, resulting in a circuit diagram as shown in Fig. 3.16.
Applying KVL gives
10

2+3
)
)

Vab2

Figure 3.15

+ Vab2 = 0
4Vab2 = 20
Vab2

= 5V

Figure 3.16

Circuit Theorems

167

According to superposition principle,


Vab

= Vab1 + Vab2
= 1 + 5 = 6V

EXAMPLE

3.6

Use the principle of superposition to solve for vx in the circuit of Fig. 3.17.

Figure 3.17

SOLUTION

According to the principle of superposition,


vx

= vx 1 + vx 2

where vx1 is produced by 6A source alone in the circuit and vx2 is produced solely by 4A current
source.
To find vx1 , deactivate the 4A current source. This results in a circuit diagram as shown in
Fig. 3.18.
KCL at node x1 :
vx 1

4ix1

vx 1

But
Hence;

)
)
)

ix1
vx 1

2
vx 1

+
+

v
4 x21

vx 1

8
2vx1

vx 1

4vx1 + vx1
vx 1

=6
=

vx 1

=6
=6

2vx1 = 48
=

48
= 16V
3

Figure 3.18

168

Network Theory

To find vx2 , deactivate the 6A current source, resulting in a circuit diagram as shown in Fig.
3.19.
KCL at node x2 :
( 4ix2 )
=4
8
2
vx 2
vx + 4ix2
+ 2
=4
8
2

vx 2

v x2

(3.3)

Applying KVL along dotted path, we get


vx 2

vx 2

+ 4ix2
2ix2

or

2ix2 = 0
ix2

vx 2

(3.4)

Substituting equation (3.4) in equation (3.3), we get

vx 2

)
)
)
)

vx 2

+4

vx 2

=4

2
vx 2

2vx2

vx 2
vx 2

8
vx 2

vx 2

=4
=4

4vx2 = 32
32
vx 2 =
V
3

Hence, according to the superposition principle,


vx

EXAMPLE

= vx 1 + vx 2
32
= 16
= 5:33V
2

Figure 3.19

3.7

Which of the source in Fig. 3.20 contributes most of the power dissipated in the 2 resistor ?
The least ? What is the power dissipated in 2 resistor ?

Figure 3.20

Circuit Theorems

169

SOLUTION

The Superposition theorem cannot be used to identify the individual contribution of each source
to the power dissipated in the resistor. However, the superposition theorem can be used to find the
total power dissipated in the 2 resistor.

Figure 3.21

According to the superposition principle,


i1

= i01 + i02

where i01 = Contribution to i1 from 5V source alone.


and i02 = Contribution to i1 from 2A source alone.
Let us first find i01 . This needs the deactivation of 2A source. Refer to Fig. 3.22.
5
= 1:22A
2 + 2:1
Similarly to find i02 we have to disable the 5V source by shorting it.
0

i1

Referring to Fig. 3.23, we find that


0

i2

Figure 3.22

2 2:1
=
2 + 2:1

1:024 A

Figure 3.23

170

Network Theory

Total current,
i1

= i01 + i02
= 1:22

1:024

= 0:196 A
Thus;

P2

= (0:196)2

2

= 0:0768 Watts
= 76:8 mW
EXAMPLE

3.8

Find the voltage V1 using the superposition principle. Refer the circuit shown in Fig.3.24.

Figure 3.24

SOLUTION

According to the superposition principle,


V1

= V10 + V100

where V10 is the contribution from 60V source alone and V100 is the contribution from 4A current
source alone.
To find V10 , the 4A current source is opened, resulting in a circuit as shown in Fig. 3.25.

Figure 3.25

Circuit Theorems

171

Applying KVL to the left mesh:


30ia

60 + 30 (ia

ib )

Also

ib

=0

(3.5)

0:4iA

0:4 (

ia )

= 0:4ia

(3.6)

Substituting equation (3.6) in equation (3.5), we get

60 + 30ia

30 0:4ia = 0
60
ia =
= 1:25A
48
1:25
ib = 0:4ia = 0:4

30ia

= 0:5A
0

Hence;

V1

= (ia

ib )

 30

= 22:5 V
To find, V100 , the 60V source is shorted as shown in Fig. 3.26.

Figure 3.26

Applying KCL at node a:


Va

Va

20
30Va

00

V1

=4
10
20V100 = 800

(3.7)

Applying KCL at node b:


00

V1

30
Also;

Va

= 20ia
V1

Va

V1

10

ib

= 0:4ib
=

Va

20
0:4Va
+
=
30
10
20
7:2Va + 8V100 = 0
00

Hence;

00

00

V1

Va

(3.8)

172

Network Theory

Solving the equations (3.7) and (3.8), we find that


00

V1

Hence

V1

= 60V
= V10 + V100
= 22:5 + 60 = 82:5V

EXAMPLE

3.9

(a) Refer to the circuit shown in Fig. 3.27. Before the 10 mA current source is attached to
terminals x y , the current ia is found to be 1.5 mA. Use the superposition theorem to find
the value of ia after the current source is connected.
(b) Verify your solution by finding ia , when all the three sources are acting simultaneously.

Figure 3.27
SOLUTION

According to the principle of superposition,


ia

= ia1 + ia2 + ia3

where ia1 , ia2 and ia3 are the contributions to ia from 20V source, 5 mA source and 10 mA source
respectively.
As per the statement of the problem,
ia1

+ ia2 = 1:5 mA

To find ia3 , deactivate 20V source and the 5 mA source. The resulting circuit diagram is
shown in Fig 3.28.
10mA 2k
= 1 mA
ia3 =
18k + 2k
Hence, total current

ia

= ia1 + ia2 + ia3


= 1:5 + 1 = 2:5 mA

Circuit Theorems

173

Figure 3.28

(b) Refer to Fig. 3.29


KCL at node y:
Vy

18

103

Vy

20
= (10+5) 10
103

Solving, we get
Hence;

ia

= 45V:
45
=
3
10
18 103
= 2:5 mA
Vy

Vy

18

Figure 3.29

3.2

Thevenins theorem

In section 3.1, we saw that the analysis of a circuit may be greatly reduced by the use of superposition principle. The main objective of Thevenins theorem is to reduce some portion of a
circuit to an equivalent source and a single element. This reduced equivalent circuit connected to
the remaining part of the circuit will allow us to find the desired current or voltage. Thevenins
theorem is based on circuit equivalence. A circuit equivalent to another circuit exhibits identical
characteristics at identical terminals.

Figure 3.30 A Linear two terminal network

Figure 3.31 The Thevenins equivalent circuit

According to Thevenins theorem, the linear circuit of Fig. 3.30 can be replaced by the one
shown in Fig. 3.31 (The load resistor may be a single resistor or another circuit). The circuit to
the left of the terminals x y in Fig. 3.31 is known as the Thevenins equivalent circuit.

174

Network Theory

The Thevenins theorem may be stated as follows:


A linear twoterminal circuit can be replaced by an equivalent circuit consisting of a
voltage source Vt in series with a resistor Rt , Where Vt is the opencircuit voltage at the terminals and Rt is the input or equivalent resistance at the terminals when the independent sources
are turned off or Rt is the ratio of opencircuit voltage to the shortcircuit current at the
terminal pair.
Action plan for using Thevenins theorem :
1. Divide the original circuit into circuit A and circuit B .

In general, circuit B is the load which may be linear or non-linear. Circuit A is the balance of
the original network exclusive of load and must be linear. In general, circuit A may contain
independent sources, dependent sources and resistors or other linear elements.

2. Separate the circuit A from circuit B .


3. Replace circuit A with its Thevenins equivalent.
4. Reconnect circuit B and determine the variable of interest (e.g. current i or voltage v ).

Procedure for finding Rt :


Three different types of circuits may be encountered in determining the resistance, Rt :
(i) If the circuit contains only independent sources and resistors, deactivate the sources and find
Rt by circuit reduction technique. Independent current sources, are deactivated by opening
them while independent voltage sources are deactivated by shorting them.

Circuit Theorems

175

(ii) If the circuit contains resistors, dependent and independent sources, follow the instructions
described below:
(a) Determine the open circuit voltage voc with the sources activated.
(b) Find the short circuit current isc when a short circuit is applied to the terminals a
(c)

Rt

voc
isc

(iii) If the circuit contains resistors and only dependent sources, then
(a)

voc

= 0 (since there is no energy source)

(b) Connect 1A current source to terminals


a
b and determine vab .
(c)

Rt

vab

Figure 3.32

For all the cases discussed above, the Thevenins equivalent circuit is as shown in Fig. 3.32.
EXAMPLE

3.10

Using the Thevenins theorem, find the current i through R = 2 . Refer Fig. 3.33.

Figure 3.33
SOLUTION

Figure 3.34

176

Network Theory

Since we are interested in the current i through R, the resistor R is identified as circuit B and
the remainder as circuit A. After removing the circuit B, circuit A is as shown in Fig. 3.35.

Figure 3.35

To find Rt , we have to deactivate the independent voltage source. Accordingly, we get the
circuit in Fig. 3.36.
Rt

jj

= (5 20 ) + 4
=

5 20
+4=8
5 + 20

Rt

Referring to Fig. 3.35,


50 + 25I = 0
Hence

Vab

= 2A

Figure 3.36

= Voc = 20(I ) = 40V

Thus, we get the Thevenins equivalent circuit which is as shown in Fig.3.37.

Figure 3.37

Figure 3.38

Reconnecting the circuit B to the Thevenins equivalent circuit as shown in Fig. 3.38, we get
i

40
= 4A
2+8

Circuit Theorems

EXAMPLE

177

3.11

(a) Find the Thevenins equivalent circuit with respect to terminals a b for the circuit shown
in Fig. 3.39 by finding the open-circuit voltage and the shortcircuit current.
(b) Solve the Thevenin resistance by removing the independent sources. Compare your result
with the Thevenin resistance found in part (a).

Figure 3.39
SOLUTION

Figure 3.40

(a) To find Voc :


Apply KCL at node 2 :
V2

)
Hence;

30
40
V2 = 60 Volts

60 + 20

Voc

V2

 V 60 0 

=I

1:5 = 0

60
60 + 20
60
= 60
= 45 V
80

178

Network Theory

To find isc :

Applying KCL at node 2:

)
Therefore;

V2

20

30

V2

1:5 = 0

40
V2 = 30V

isc

Rt

V2

20
Voc
isc

= 1:5A
=

45
1:5

= 30
Figure 3.40 (a)

The Thevenin equivalent circuit with respect to the terminals a b is as shown in Fig. 3.40(a).
(b) Let us now find Thevenin resistance Rt by deactivating all the independent sources,

Rt

Rt

Rt

jj

= 60 (40 + 20)
60
=
= 30 (veried)
2

It is seen that, if only independent sources are present, it is easy to find Rt by deactivating all
the independent sources.

Circuit Theorems

EXAMPLE

3.12

Find the Thevenin equivalent for the circuit shown in Fig. 3.41 with respect to terminals a

Figure 3.41
SOLUTION

To find Voc = Vab :


Applying KVL around the mesh of
Fig. 3.42, we get

20 + 6i

2i + 6i = 0
i

= 2A

Since there is no current flowing in


10 resistor, Voc = 6i = 12 V
To find Rt : (Refer Fig. 3.43)
Since both dependent and independent sources are present, Thevenin resistance is found using the relation,

Figure 3.42

Rt

voc
isc

Applying KVL clockwise for mesh 1 :


20 + 6i1

2i + 6 (i1
12i1

Since i = i1

i2 ,

i2 )

=0

6i2 = 20 + 2i

we get

12i1

6i2 = 20 + 2 (i1

10i1

4i2 = 20

i2 )

Applying KVL clockwise for mesh 2 :

10i2 + 6 (i2

i1 )

=0

6i1 + 16i2 = 0

Figure 3.43

b.

179

180

Network Theory

Solving the above two mesh equations, we get

EXAMPLE

i2

Rt

120
A
136
voc
isc

isc

= i2 =

120
A
136

12
= 13:6
120
136

3.13

Find Vo in the circuit of Fig. 3.44 using Thevenins theorem.

Figure 3.44
SOLUTION

To find Voc :
Since we are interested in the voltage across 2 k resistor, it is removed from the circuit of
Fig. 3.44 and so the circuit becomes as shown in Fig. 3.45.

Figure 3.45

By inspection,
Applying KVL to mesh 2 :

i1

= 4 mA

12 + 6

12 + 6

 10

 10

i2

(i2

 10

 + 3  103 i2 = 0
3
3

i1 )

+3

 10

i2

=0

Circuit Theorems

Solving, we get

i2

181

= 4 mA

! a b ! 3 k, we get
+
3  10 = 0
= 4  10 + 3  10
= 4  10  4  10 + 3  10  4  10

Applying KVL to the path 4 k


4

 10

i1

Voc

Voc

i2

i1

i2

= 28V

To find Rt :
Deactivating all the independent sources, we get the circuit diagram shown in Fig. 3.46.

Figure 3.46

jj

= Rab = 4 k + (6 k 3 k) = 6 k

Rt

Hence, the Thevenin equivalent circuit is as shown in Fig. 3.47.

Figure 3.47

Figure 3.48

If we connect the 2 k resistor to this equivalent network, we obtain the circuit of Fig. 3.48.
Vo

 10 
28
 2  10
=
(6 + 2)  10

=i 2

EXAMPLE

= 7V

3.14

The wheatstone bridge in the circuit shown in Fig. 3.49 (a) is balanced when R2 = 1200 . If the
galvanometer has a resistance of 30 , how much current will be detected by it when the bridge
is unbalanced by setting R2 to 1204 ?

182

Network Theory

Figure 3.49(a)
SOLUTION

To find Voc :
We are interested in the galavanometer current. Hence, it is removed from the circuit of Fig.
3.49 (a) to find Voc and we get the circuit shown in Fig. 3.49 (b).
120
120
=
A
i1 =
900 + 600
1500
120
120
i2 =
=
A
1204 + 800
2004
Applying KVL clockwise along the path
1204
b
a
900 , we get

1204i2

Vt

900i1 = 0

900i1
120
900
= 1204
2004
= 95:8 mV

Vt

= 1204i2

120
 1500
Figure 3.49(b)

To find Rt :
Deactivate all the independent sources and look into the terminals
Thevenins resistance.

Figure 3.49(c)

Figure 3.49(d)

to determine the

Circuit Theorems

Rt

jj

iG

jj

95:8 10 3
840:64 + 30

Figure 3.50

= 110:03 A

3.15

For the circuit shown in Fig. 3.51, find the Thevenins equivalent circuit between terminals

Figure 3.51
SOLUTION

With ab shorted, let Isc = I . The circuit after


transforming voltage sources into their equivalent current sources is as shown in Fig 3.52.
Writing node equations for this circuit,
0:1 Vc + I = 3

At a :

0:2Va

At c :

0:1Va + 0:3 Vc

At b :

183

= Rab = 600 900 + 800 1204


900 600 1204 800
=
+
1500
2004
= 840:64

Hence, the Thevenin equivalent circuit consists of the


95.8 mV source in series with 840.64 resistor. If we
connect 30 resistor (galvanometer resistance) to this
equivalent network, we obtain the circuit in Fig. 3.50.

EXAMPLE

0:1 Vb = 4

0:1Vc + 0:2 Vb

=1

As the terminals a and b are shorted Va =


and the above equations become

Vb

Figure 3.52

a and b.

184

Network Theory

0:2Va

0:1 Vc + I = 3

0:2Va + 0:3 Vc = 4
0:2Va

0:1 Vc

1=1

Solving the above equations, we get the short circuit current, I = Isc = 1 A.
Next let us open circuit the terminals a and b and this makes I = 0. And the node equations
written earlier are modified to
0:2Va

0:1 Vc = 3

0:1Va + 0:3 Vc

0:1 Vb = 4

0:1Vc + 0:2 Vb = 1
Solving the above equations, we get
Va

= 30V and Vb = 20V

Hence, Vab = 30

20 = 10 V = Voc = Vt
10
Therefore Rt =
=
= 10
Isc
1
The Thevenins equivalent is as shown in Fig 3.53
Voc

Figure 3.53

EXAMPLE

3.16

Refer to the circuit shown in Fig. 3.54. Find the Thevenin equivalent circuit at the terminals a

b.

Figure 3.54
SOLUTION

To begin with let us transform 3 A current source and 10 V voltage source. This results in a
network as shown in Fig. 3.55 (a) and further reduced to Fig. 3.55 (b).

Circuit Theorems

185

Figure 3.55(a)

Again transform the 30 V source and following the reduction procedure step by step from
Fig. 3.55 (b) to 3.55 (d), we get the Thevenins equivalent circuit as shown in Fig. 3.56.

Figure 3.55(b)

Figure 3.55(d)

Figure 3.55(c)

Figure 3.56 Thevenin equivalent


circuit

EXAMPLE

3.17

Find the Thevenin equivalent circuit as seen from the terminals a


shown in Fig. 3.57.

b.

Refer the circuit diagram

186

Network Theory

Figure 3.57

SOLUTION

Since the circuit has no independent sources, i = 0 when the terminals a b are open. Therefore, Voc = 0.
The onus is now to find Rt . Since Voc = 0 and isc = 0, Rt cannot be determined from
Rt

Voc
isc

. Hence, we choose to connect a source of 1 A at the terminals a

as shown in Fig.

3.58. Then, after finding Vab , the Thevenin resistance is,


Rt

KCL at node a :

2i

Va

5
Also;

Va

Hence;

2
5

Va
10

+
=
+

Vab

Va

1=0

10
Va

10
Va

10
50
Va =
V
13

1=0

50

1
13
Alternatively one could find Rt by connecting a 1V source at the terminals a b and then find
1
. The concept of finding Rt by connecting a 1A source
the current from b to a. Then Rt =
Hence;

Rt

Va

iba

between the terminals a b may also be used for circuits containing independent sources. Then
set all the independent sources to zero and use 1A source at the terminals a b to find Vab and
Vab

.
hence, Rt =
1
For the present problem, the Thevenin equivalent circuit as seen between the terminals a
is shown in Fig. 3.58 (a).

Figure 3.58

Figure 3.58 (a)

Circuit Theorems

EXAMPLE

187

3.18

Determine the Thevenin equivalent circuit between the terminals a

for the circuit of Fig. 3.59.

Figure 3.59

SOLUTION

As there are no independent sources in the circuit, we get Voc = Vt = 0:


To find Rt , connect a 1V source to the terminals a b and measure the current I that flows
from b to a. (Refer Fig. 3.60 a).
1

Rt =
I

Figure 3.60(a)

Applying KCL at node a:


I

Since;

Vx

we get,

Hence;

= 0:5Vx +

Vx

= 1V
= 0:5 +

1
= 0:75 A
4

Figure 3.60(b)

1
Rt =
= 1:33
0:75

The Thevenin equivalent circuit is shown in 3.60(b).


Alternatively, sticking to our strategy, let us connect 1A current source between the terminals
a

and then measure Vab (Fig. 3.60 (c)). Consequently, Rt =

Vab

= Vab :

188

Network Theory

Applying KCL at node a:

0:5Vx +
Hence

Rt

Vx

4
Vab

=1
=

Vx

Vx

= 1:33V

= 1:33

The corresponding Thevenin equivalent


circuit is same as shown in Fig. 3.60(b)

3.3

Figure 3.60(c)

Nortons theorem

An American engineer, E.L. Norton at Bell Telephone Laboratories, proposed a theorem similar
to Thevenins theorem.
Nortons theorem states that a linear two-terminal network can be replaced by an
equivalent circuit consisting of a current source iN in parallel with resistor RN , where iN
is the short-circuit current through the terminals and RN is the input or equivalent resistance
at the terminals when the independent sources are turned off. If one does not wish to turn off
the independent sources, then RN is the ratio of open circuit voltage to shortcircuit current
at the terminal pair.

Figure 3.61(a) Original circuit

Figure 3.61(b) Nortons equivalent circuit

Figure 3.61(b) shows Nortons equivalent circuit as seen from the terminals a b of the
original circuit shown in Fig. 3.61(a). Since this is the dual of the Thevenin circuit, it is clear that
voc
. In fact, source transformation of Thevenin equivalent circuit leads to
RN = Rt and iN =
Rt

Nortons equivalent circuit.


Procedure for finding Nortons equivalent circuit:
(1) If the network contains resistors and independent sources, follow the instructions below:
(a) Deactivate the sources and find RN by circuit reduction techniques.
(b) Find iN with sources activated.
(2) If the network contains resistors, independent and dependent sources, follow the steps given
below:
(a) Determine the short-circuit current iN with all sources activated.

Circuit Theorems

189

(b) Find the open-circuit voltage voc .


(c)

Rt

= RN =

voc
iN

(3) If the network contains only resistors and dependent sources, follow the procedure
described below:
(a) Note that iN = 0.
(b) Connect 1A current source to the terminals a
(c)

Rt

vab

and find vab .

Note: Also, since vt = voc and iN = isc


Rt

voc
isc

= RN

The opencircuit and shortcircuit test are sufficient to find any Thevenin or Norton equivalent.
3.3.1 PROOF OF THEVENINS AND NORTONS THEOREMS

The principle of superposition is employed to provide the proof of Thevenins and Nortons
theorems.
Derivation of Thevenins theorem:

Let us consider a linear circuit having two accessible terminals x y and excited by an external
current source i. The linear circuit is made up of resistors, dependent and independent sources. For
the sake of simplified analysis, let us assume that the linear circuit contains only two independent
voltage sources v1 and v2 and two independent current sources i1 and i2 . The terminal voltage v
may be obtained, by applying the principle of superposition. That is, v is made up of contributions
due to the external source and independent sources within the linear network.
Hence;

= a0 i + a1 v1 + a2 v2 + a3 i1 + a4 i2

(3.9)

= a0 i + b0
where

b0

(3.10)

= a1 v1 + a2 v2 + a3 i1 + a4 i2
= contribution to the terminal voltage v by
independent sources within the linear network.

Let us now evaluate the values of constants a0 and b0 .


(i) When the terminals x and y are opencircuited,
this fact in equation 3.10, we find that b0 = vt .

= 0 and

voc

vt .

Making use of

190

Network Theory

(ii) When all the internal sources are deactivated,


become
v = a0 i = Rt i

b0

= 0. This enforces equation 3.10 to

a0

= Rt
Rt

Vt

Figure 3.62

Figure 3.63 Thevenins equivalent circuit of Fig. 3.62

Current-driven circuit

where Rt is the equivalent resistance of the linear network as viewed from the terminals x y .
Also, a0 must be Rt in order to obey the ohms law. Substuting the values of a0 and b0 in equation
3.10, we find that
v = Rt i + v1
which expresses the voltage-current relationship at terminals
Thus, the two circuits of Fig. 3.62 and 3.63 are equivalent.

of the circuit in Fig. 3.63.

Derivation of Nortons theorem:

Let us now assume that the linear circuit described earlier is driven by a voltage source v as shown
in Fig. 3.64.
The current flowing into the circuit can be obtained by superposition as
i

= c0 v + d 0

(3.11)

where c0 v is the contribution to i due to the external voltage source v and d0 contains the contributions to i due to all independent sources within the linear circuit. The constants c0 and d0 are
determined as follows :
(i) When terminals x y are short-circuited, v =
0 and i = isc . Hence from equation (3.11),
we find that i = d0 = isc , where isc is the
short-circuit current flowing out of terminal x,
which is same as Norton current iN
Thus,

d0

iN

Figure 3.64
Voltage-driven circuit

(ii) Let all the independent sources within the linear network be turned off, that is d0 = 0. Then,
equation (3.11) becomes
i = c0 v

Circuit Theorems

191

For dimensional validity, c0 must have the


dimension of conductance. This enforces c0 =
1
where Rt is the equivalent resistance of the
Rt

linear network as seen from the terminals x


Thus, equation (3.11) becomes
i

=
=

1
Rt

1
Rt

isc

iN

y.

Figure 3.65 Nortons equivalent of


voltage driven circuit

This expresses the voltage-current relationship at the terminals x y of the circuit in Fig.
(3.65), validating that the two circuits of Figs. 3.64 and 3.65 are equivalents.
EXAMPLE

3.19

Find the Norton equivalent for the circuit of Fig. 3.66.

Figure 3.66
SOLUTION

As a first step, short the terminals a b. This


results in a circuit diagram as shown in Fig. 3.67.
Applying KCL at node a, we get
0

24
4

3 + isc = 0

isc

= 9A

To find RN , deactivate all the independent


sources, resulting in a circuit diagram as shown
in Fig. 3.68 (a). We find RN in the same way as
Rt in the Thevenin equivalent circuit.
4 12
RN =
=3
4 + 12

Figure 3.67

192

Network Theory

Figure 3.68(a)

Figure 3.68(b)

Thus, we obtain Nortion equivalent circuit as shown in Fig. 3.68(b).


EXAMPLE

3.20

Refer the circuit shown in Fig. 3.69. Find the value of ib using Norton equivalent circuit. Take
R = 667 .

Figure 3.69
SOLUTION

Since we want the current flowing through R, remove


R from the circuit of Fig. 3.69. The resulting circuit
diagram is shown in Fig. 3.70.
To find iac or iN referring Fig 3.70(a) :
0
= 0A
1000
12
isc =
A = 2 mA
6000
ia

Figure 3.70

Figure 3.70(a)

Circuit Theorems

To find RN :
The procedure for finding RN is same that of Rt
in the Thevenin equivalent circuit.
Rt

= RN =

voc
isc

To find voc , make use of the circuit diagram shown


in Fig. 3.71. Do not deactivate any source.
Applying KVL clockwise, we get

)
)
Therefore;

12 + 6000ia + 2000ia + 1000ia = 0


4
ia =
A
3000
4
voc = ia
1000 = V
3
4
voc
3
RN =
=
= 667
isc
2 10 3

Figure 3.71

The Norton equivalent circuit along with resistor R is as shown below:


ib

isc

2mA
= 1mA
2

Figure : Norton equivalent circuit with load R

EXAMPLE

3.21

Find Io in the network of Fig. 3.72 using Nortons theorem.

Figure 3.72

193

194

Network Theory

SOLUTION

We are interested in Io , hence the 2 k resistor is removed from the circuit diagram of Fig. 3.72.
The resulting circuit diagram is shown in Fig. 3.73(a).

Figure 3.73(a)

Figure 3.73(b)

To find iN or isc :
Refer Fig. 3.73(b). By inspection, V1 = 12 V
Applying KCL at node V2 :
V2

V1

6 k

V2

2 k

V2

V1

3 k

=0

Substituting V1 = 12 V and solving, we get


V2

= 6V

isc

V1

V2

3 k

V1

4 k

= 5 mA

To find RN :
Deactivate all the independent sources (refer Fig. 3.73(c)).

Figure 3.73(c)

Figure 3.73(d)

Circuit Theorems

195

Referring to Fig. 3.73 (d), we get


RN

jj

jj

= Rab = 4 k [3 k + (6 k 2 k)] = 2:12 k

Hence, the Norton equivalent circuit


along with 2 k resistor is as shown in
Fig. 3.73(e).
Io

EXAMPLE

isc
R

RN

+ RN

= 2:57mA
Figure 3.73(e)

3.22

Find Vo in the circuit of Fig. 3. 74.

Figure 3.74

SOLUTION

Since we are interested in Vo , the voltage across 4 k resistor, remove this resistance from the
circuit. This results in a circuit diagram as shown in Fig. 3.75.

Figure 3.75

196

Network Theory

To find isc , short the terminals a

Circuit Theorems

Constraint equation :
i2

i1

KVL around supermesh :


4+2
KVL around mesh 3 :
8

 10 (
3

i3

= 4mA

 10

i2 )

+2

i1

197

(3.12)

 10

+4

i2

=0

i3

i1 )

=0

isc

i1 )

=0

 10 (
3

(3.13)

Since i3 = isc , the above equation becomes,


8

 10 (
3

isc

i2 )

+2

 10 (
3

(3.14)

Solving equations (3.12), (3.13) and (3.14) simultaneously, we get isc = 0:1333 mA.
To find RN :
Deactivate all the sources in Fig. 3.75. This yields a circuit diagram as shown in Fig. 3.76.

Figure 3.76

RN

jj

= 6 k 10 k
6 10
=
= 3:75 k
6 + 10

Hence, the Norton equivalent circuit is as shown


in Fig 3.76 (a).
To the Norton equivalent circuit, now connect the
4 k resistor that was removed earlier to get the
network shown in Fig. 3.76(b).

Figure 3.76(a)

198

Network Theory

Vo

= isc (RN
= isc

jj

R)

RN R
RN

+R

= 258 mV

Figure 3.76(b) Norton equivalent circuit with R = 4 k

EXAMPLE

3.23

Find the Norton equivalent to the left of the terminals a

for the circuit of Fig. 3.77.

Figure 3.77

SOLUTION

To find isc :

Note that vab = 0 when the terminals a

are short-circuited.

5
= 10 mA
500
Therefore, for the righthand portion of the circuit, isc =

Then

10i =

100 mA.

Circuit Theorems

199

To find RN or Rt :

Writing the KVL equations for the left-hand mesh, we get


5 + 500i + vab = 0

(3.15)

Also for the right-hand mesh, we get


vab

Therefore

=
=

25(10i) =

250i

vab

250
Substituting i into the mesh equation (3.15), we get

)
RN

5 + 500

vab

+ vab = 0

250

= 5V
5
= 50
0:1
vab

= Rt 

voc
isc

vab
isc

The Norton equivalent circuit is shown in


Fig 3.77 (a).
Figure 3.77 (a)

EXAMPLE

3.24

Find the Norton equivalent of the network shown in Fig. 3.78.

Figure 3.78

200

Network Theory

SOLUTION

Since there are no independent sources present in the network of Fig. 3.78, iN = isc = 0.
To find RN , we inject a current of 1A between the terminals a b. This is illustrated in
Fig. 3.79.

Figure 3.79

Figure 3.79(a)

Norton

equivalent circuit

KCL at node 1:
1=

v1

100
0:03v1

KCL at node 2:

v2

200

v2

v1

v2

50
0:02v2 = 1

v1

+ 0:1v1 = 0
50
0:08v1 + 0:025v2 = 0

Solving the above two nodal equations, we get


= 10:64 volts

voc = 10:64 volts


10:64
Hence;
R N = Rt =
=
= 10:64
1
1
Norton equivalent circuit for the network shown in Fig. 3.78 is as shown in Fig. 3.79(a).
v1

voc

EXAMPLE

3.25

Find the Thevenin and Norton equivalent circuits for the network shown in Fig. 3.80 (a).

Figure 3.80(a)

Circuit Theorems

201

SOLUTION

To find Voc :
Performing source transformation on 5A current source, we get the circuit shown in
Fig. 3.80 (b).
Applying KVL around Left mesh :

50 + 2ia

20 + 4ia = 0
70
ia =
A
6

Applying KVL around right mesh:


20 + 10ia + Voc

4ia = 0
Voc

90 V
Figure 3.80(b)

To find isc (referring Fig 3.80 (c)) :


KVL around Left mesh :
50 + 2ia

20 + 4 (ia

isc )

=0

6ia 4isc = 70
KVL around right mesh :
4 (isc

ia )

+ 20 + 10ia = 0
6ia + 4isc =

20
Figure 3.80(c)

Solving the two mesh equations simultaneously, we get isc = 11:25 A


voc
90
=
=8
Hence, Rt = RN =
isc
11:25
Performing source transformation on Thevenin equivalent circuit, we get the norton equivalent
circuit (both are shown below).

Thevenin equivalent circuit

Norton equivalent circuit

202

Network Theory

EXAMPLE

3.26

If an 8 k load is connected to the terminals of the


network in Fig. 3.81, VAB = 16 V. If a 2 k load is
connected to the terminals, VAB = 8V. Find VAB if a
20 k load is connected across the terminals.
SOLUTION

Figure 3.81

Applying KVL around the mesh, we get (Rt + RL ) I = Voc


If

RL

= 2 k;

If

RL

= 10 k;

)
= 6 mA )

= 10 mA
I

Voc

= 20 + 0:01Rt

Voc

= 60 + 0:006Rt

Solving, we get Voc = 120 V, Rt = 10 k.


If

3.4

RL

= 20 k;

Voc

(RL + Rt )

(20

120
+ 10

103

 10 ) = 4 mA
3

Maximum Power Transfer Theorem

In circuit analysis, we are some times interested


in determining the maximum power that a circuit
can supply to the load. Consider the linear circuit
A as shown in Fig. 3.82.
Circuit A is replaced by its Thevenin equivalent
circuit as seen from a and b (Fig 3.83).
We wish to find the value of the load RL such that
the maximum power is delivered to it.
The power that is delivered to the load is given by

=i


RL

Vt
Rt

+ RL

Figure 3.82 Circuit A with load RL

2
RL

(3.16)

Circuit Theorems

203

Assuming that Vt and Rt are fixed for a given source, the maximum power is a function of
In order to determine the value of RL that maximizes p, we differentiate p with respect to
RL and equate the derivative to zero.

RL .

"

dp

= Vt2

dRL

which yields

RL

(Rt + RL )2 2 (Rt + RL )
=0
(RL + Rt )2

= Rt

(3.17)

To confirm that equation (3.17) is a maximum,


2

it should be shown that

d p

2
dRL

<

0. Hence, maxi-

mum power is transferred to the load when RL is


equal to the Thevenin equivalent resistance Rt .
The maximum power transferred to the load is
obtained by substituting RL = Rt in equation
3.16.
Accordingly,
2

Pmax

Vt R L

(2RL )2

Figure 3.83 Thevenin equivalent circuit


is substituted for circuit A

Vt

4RL

The maximum power transfer theorem states that the maximum power delivered by a source
represented by its Thevenin equivalent circuit is attained when the load RL is equal to the
Thevenin resistance Rt .
EXAMPLE

3.27

Find the load RL that will result in maximum power delivered to the load for the circuit of Fig.
3.84. Also determine the maximum power Pmax .

Figure 3.84

SOLUTION

Disconnect the load resistor RL . This results in a circuit diagram as shown in Fig. 3.85(a).
Next let us determine the Thevenin equivalent circuit as seen from a b.

204

Network Theory

180
= 1A
150 + 30
i = 150 V
Voc = Vt = 150
i

To find Rt , deactivate the 180 V source. This results in the


circuit diagram of Fig. 3.85(b).
Rt

jj

= Rab = 30 150
30 150
=
= 25
30 + 150

Figure 3.85(a)

The Thevenin equivalent circuit connected to the


load resistor is shown in Fig. 3.86.
Maximum power transfer is obtained when
RL = Rt = 25 :
Then the maximum power is
(150)2
4RL
4 25
= 2:25 Watts
The Thevenin source Vt actually provides a total
power of
2

Pmax

Pt

Vt

Figure 3.85(b)



150
= 150 
25 + 25
= 150

= 450 Watts
Thus, we note that one-half the power is dissipated in RL .
EXAMPLE

Figure 3.86

3.28

Refer to the circuit shown in Fig. 3.87. Find the value of RL for maximum power transfer. Also
find the maximum power transferred to RL .

Figure 3.87

Circuit Theorems

SOLUTION

Disconnecting RL , results in a circuit diagram as shown in Fig. 3.88(a).

Figure 3.88(a)

To find Rt , deactivate all the independent voltage sources as in Fig. 3.88(b).

Figure 3.88(b)
Rt

Figure 3.88(c)

jj

jj

= Rab = 6 k 6 k 6 k
= 2 k

To find Vt :
Refer the Fig. 3.88(d).
Constraint equation :
V3

V1

By inspection,
KCL at supernode :

= 12 V

V2

=3V
V2

V3

6k

V3

6k

12

V3

6k

V1

6k

V3

V1

V2

=0
6k
12 3
=0
6k

Figure 3.88(d)

205

206

Network Theory

)
)
)
)

V3

3 + V3

12 + V3

15 = 0
3V3 = 30
= 10

V3

= Vab = V3 = 10 V

Vt

Figure 3.88(e)

The Thevenin equivalent circuit connected to the load resistor RL is shown in Fig. 3.88(e).
Pmax

= i2 RL

Vt

2
RL

2RL
= 12:5 mW

Alternate method :
It is possible to find Pmax , without finding the Thevenin equivalent circuit. However, we have to
find Rt . For maximum power transfer, RL = Rt = 2 k. Insert the value of RL in the original
circuit given in Fig. 3.87. Then use any circuit reduction technique of your choice to find power
dissipated in RL .
Refer Fig. 3.88(f). By inspection we find that, V2 = 3 V.
Constraint equation :
V3

V1

= 12

V1

= V3

V1

V2

12

KCL at supernode :
V2

V3

6k

)
)
)
)

V3

6k
V3

V3

12
6k

3 + V3

6k
3

V3

V3

2k

V3

6k
12

=0

+
=0
2k
6k
15 + 3V3 + V3 12 = 0
6V3 = 30
V3

Hence;

V1

Pmax

V3

RL

=5 V

25
= 12:5 mW
2k

Figure 3.88(f)

Circuit Theorems

EXAMPLE

207

3.29

Find RL for maximum power transfer and the maximum power that can be transferred in the
network shown in Fig. 3.89.

Figure 3.89

SOLUTION

Disconnect the load resistor RL . This results in a circuit as shown in Fig. 3.89(a).

Figure 3.89(a)

To find Rt , let us deactivate all the independent sources, which results the circuit as shown in
Fig. 3.89(b).
Rt = Rab = 2 k + 3 k + 5 k = 10 k
For maximum power transfer RL = Rt = 10 k.
Let us next find Voc or Vt .
Refer Fig. 3.89 (c). By inspection, i1 = 2 mA & i2 = 1 mA.

208

Network Theory

Figure 3.89(b)

! 3 k ! 2 k ! , we get
) + 2k  + = 0
5k  + 3k (


1  10
2  10
+3  10
2  10
+2  10

Applying KVL clockwise to the loop 5 k

) 5  10 1  10
)
)
3

i1

i2

i2

i1

Vt

4 + Vt = 0
Vt

= 18 V:

The Thevenin equivalent circuit with load resistor RL is as shown in Fig. 3.89 (d).
18
= 0:9 mA
i =
(10 + 10) 103
Then,

Pmax

= PL = (0:9 mA)2

 10 k

= 8:1 mW

Figure 3.89(c)

EXAMPLE

Figure 3.89(d)

3.30

Find the maximum power dissipated in RL . Refer the circuit shown in Fig. 3.90.

Figure 3.90

+ Vt = 0

Circuit Theorems

209

SOLUTION

Disconnecting the load resistor RL from the original circuit results in a circuit diagram as shown
in Fig. 3.91.

Figure 3.91

As a first step in the analysis, let us find Rt . While finding Rt , we have to deactivate all the
independent sources. This results in a network as shown in Fig 3.91 (a) :

Figure 3.91(a)
Rt

jj

= Rab = [140 60 ] + 8
140 60
=
+ 8 = 50 :
140 + 60

For maximum power transfer, RL = Rt = 50 . Next step in the analysis is to find Vt .


Refer Fig 3.91(b), using the principle of
current division,
i1

i2


+
20  170
=
= 17 A
170 + 30
 = 20  30
=
=

R1

R2

R2

R1

R1

+ R2

600
= 3A
200

170 + 30

Figure 3.91(a)

210

Network Theory

Applying KVL clockwise to the loop comprising of 50


50i2

10i1 + 8

) 50(3)
)

0+

Vt

EXAMPLE

b,

we get

10 (17) + Vt = 0
Vt

= 20 V

20
= 0:2A
50 + 50
2
Pmax = iT
50 = 0:04 50 = 2 W
=

=0

The Thevenin equivalent circuit with load resistor


as shown in Fig. 3.91(c).
iT

! 10 ! 8 !

RL

is
Figure 3.91(c)

3.31

Find the value of


find Pmax .

RL

for maximum power transfer in the circuit shown in Fig. 3.92. Also

Figure 3.92
SOLUTION

Disconnecting RL from the original circuit, we get the network shown in Fig. 3.93.

Figure 3.93

Circuit Theorems

211

Let us draw the Thevenin equivalent circuit as seen from the terminals a b and then insert
the value of RL = Rt between the terminals a b. To find Rt , let us deactivate all independent
sources which results in the circuit as shown in Fig. 3.94.

Figure 3.94
Rt

= Rab

jj


=8 2
8 2
=
= 1:6
8+2
Next step is to find Voc or Vt .
By performing source transformation on the circuit shown in Fig. 3.93, we obtain the circuit
shown in Fig. 3.95.

Figure 3.95

Applying KVL to the loop made up of 20 V

20 + 10i

! 3 ! 2 ! 10 V ! 5 ! 30 V, we get
10

30 = 0
60
i =
= 6A
10

212

Network Theory

Again applying KVL clockwise to the path 2


2i 10 Vt = 0

Vt

= 2(6)

= 2i

! 10 V !

b,

we get

10

10 = 2 V

The Thevenin equivalent circuit with load resistor


RL is as shown in Fig. 3.95 (a).
Pmax

= i2T RL
2

=
EXAMPLE

Vt

4Rt

= 625 mW

Figure 3.95(a) Thevenin equivalent


circuit

3.32

Find the value of RL for maximum power transfer. Hence find Pmax .

Figure 3.96
SOLUTION

Removing RL from the original circuit gives us the circuit diagram shown in Fig. 3.97.

Figure 3.97

To find Voc :
KCL at node A :

)
Hence;

ia

0:9 + 10i0a = 0
0

ia
Voc

= 0:1 A
= 3 10i0a
=3

 10  0 1 = 3 V
:

Circuit Theorems

213

To find Rt , we need to compute isc with all independent sources activated.


KCL at node A:

)
Hence

isc

ia

00

0:9 + 10ia 00 = 0
ia

= 10ia = 10
=

= 0:1 A

01=1A

00

Rt

00

Voc

isc

3
=3
1

Hence, for maximum power transfer RL = Rt = 3 .


The Thevenin equivalent circuit with RL = 3
inserted between the terminals a b gives the network shown in Fig. 3.97(a).
iT
Pmax

3
= 0:5 A
3+3

= i2T RL

= (0:5)2

3

= 0.75 W
EXAMPLE

Figure 3.97(a)

3.33

Find the value of RL in the network shown that will achieve maximum power transfer, and determine the value of the maximum power.

Figure 3.98(a)
SOLUTION

Removing RL from the circuit of Fig. 3.98(a), we


get the circuit of Fig 3.98(b).
Applying KVL clockwise we get
12 + 2 103 i + 2Vx0 = 0

Also
Hence;

Vx

 10

 10
12
=
4  10

12 + 2
i

=1

3i

+2 1

 10

= 3 mA

3i

=0
Figure 3.98(b)

214

Network Theory

!2 !
1  10


Applying KVL to loop 1 k

Vx

Vt

a,

b
i

we get

+ 2Vx0

Vt

=0

 10 + 2 1  10
= 1  10 + 2  10

= 3  10 3  10
3

=1

=9V
To find Rt , we need to find isc . While finding isc ,
none of the independent sources must be deactivated.
Applying KVL to mesh 1:
12 + Vx 00 + 0 = 0

)
)

00

= 12

i1

= 12

Vx

 10

i1

= 12 mA

Applying KVL to mesh 2:


1

 10

i2

+ 2Vx 00 = 0

 10

i2

24

i2

24 mA

Applying KCL at node a:


isc

= i1

i2

= 12 + 24 = 36 mA
Hence;

Rt

Vt
isc

Voc
isc

9
=
36 10
= 250

For maximum power transfer, RL = Rt = 250 .


Thus, the Thevenin equivalent circuit with RL is
as shown in Fig 3.98 (c) :
9
9
=
A
250 + 250
500
2
Pmax = iT
250
iT



9 2
500
= 81 mW
=

 250

Figure 3.98 (c) Thevenin equivalent circuit

Circuit Theorems

EXAMPLE

215

3.34

The variable resistor RL in the circuit of Fig. 3.99 is adjusted untill it absorbs maximum power
from the circuit.
(a) Find the value of RL .
(b) Find the maximum power.

Figure 3.99
SOLUTION

Disconnecting the load resistor


Fig. 3.99(a).

RL

from the original circuit, we get the circuit shown in

Figure 3.99(a)

KCL at node v1 :
100

v1

v1

13i0a
+
5

v2

v1

=0

(3.18)

Constraint equations :
0

ia
v1

v2

4
0

va

100

v1

= va0
= v1

(3.19)

v2

(applying K C L at v2 )

(3.20)

(potential across 4 )

(3.21)

216

Network Theory

From equations (3.20) and (3.21), we have


v2

)
)
)

v1

4
v2

v1

5v1

= v1

v2

= 4v1

4v2

5v2 = 0
v1

(3.22)

= v2

Making use of equations (3.19) and (3.22) in (3.18), we get


100

v1

)
)
)
)

13

v2

v1 )

(100
2


100) + 2

5 (v1

13

v1

500 + 2v1

5v1

13

(100

v1

v1

4
v1 )

=0

=0
2
100 + 13v1 = 0

20v1 = 1800

Hence;
We know that,

v1

= 90 Volts

vt

= v2 = v1 = 90 Volts

Rt

voc
isc

vt

isc

The short circuit current is calculated using the circuit shown below:

00

Here
Applying KCL at node v1 :

ia

100

v1

100

v1

v1

100

v1

2


13ia
0
v1
+
=0
5
4
(100 v1 )
13
v1
2
+
=0
5
4

v1

Circuit Theorems

Solving we get v1 = 80 volts = va00


Applying KCL at node a :
0

v1

+ isc = va00
v1

+ va00
4
80
=
+ 80 = 100 A
4
voc
vt
=
Rt =
isc

Hence;

isc

isc

90
= 0:9
=
100
Hence for maximum power transfer,
RL

= Rt = 0:9

The Thevenin equivalent circuit with RL = 0:9


is as shown.
90
90
=
0:9 + 0:9
1:8
2
Pmax = it
0:9
it

EXAMPLE

 90 2
1:8

 0 9 = 2250 W
:

3.35

Refer to the circuit shown in Fig. 3.100 :


(a) Find the value of RL for maximum power transfer.
(b) Find the maximum power that can be delivered to RL .

Figure 3.100

217

218

Network Theory

SOLUTION

Removing the load resistor RL , we get the circuit diagram shown in Fig. 3.100(a). Let us proceed
to find Vt .

Figure 3.100(a)

Constraint equation :
0

ia

= i1

i3

KVL clockwise to mesh 1 :

200 + 1 (i1

i2 )

+ 20 (i1
25i1

i3 )

+ 4i1 = 0

i2

20i3 =

200

KVL clockwise to mesh 2 :

)
)

14i0a + 2 (i2
14 (i1

i3 )

+ 2 (i2

i3 )

+ 1 (i2

i1 )

=0

i3 )

+ 1 (i2

i1 )

=0

13i1 + 3i2

16i3 = 0

KVL clockwise to mesh 3 :

2 (i3

i2 )

100 + 3i3 + 20 (i3


20i1

i1 )

=0

2i2 + 25i3 = 100

Solving the mesh equations, we get


i1

2:5A; i3 = 5A

Applying KVL clockwise to the path comprising of a

Vt

20i0a = 0
Vt

= 20i0a
= 20 (i1

i3 )

= 20 ( 2:5
=

150 V

5)

! 20 , we get

Circuit Theorems

Next step is to find Rt .


Rt

When terminals a

Voc

isc

Vt
isc

are shorted, i00a = 0. Hence, 14 i00a is also zero.

KVL clockwise to mesh 1 :

200 + 1 (i1

i2 )

+ 4i1 = 0

5i1

i2

200

KVL clockwise to mesh 2 :

2 (i2

i3 )

+ 1 (i2

i1

+ 3i2

i1 )

=0

2i3 = 0

KVL clockwise to mesh 3 :

100 + 3i3 + 2 (i3

i2 )

=0

2i2 + 5i3 = 100

219

220

Network Theory

Solving the mesh equations, we find that


i1

40A;

i3

= 20A;

= i1

i3 = 60A
150
Rt =
=
= 2:5
isc
60
For maximum power transfer, RL = Rt = 2:5 . The Thevenin equivalent circuit with RL is
as shown below :
isc

Vt

Pmax

= i21 RL

150
=
2:5 + 2:5
= 2250 W
EXAMPLE

2

25
:

3.36

A practical current source provides 10 W to a 250 load and 20 W to an 80 load. A resistance


RL , with voltage vL and current iL , is connected to it. Find the values of RL , vL and iL if
(a) vL iL is a maximum, (b) vL is a maximum and (c) iL is a maximum.
SOLUTION

Load current calculation:

10
250
=r
200 mA
20
20W to 80 corresponds to iL =
80
= 500 mA
Using the formula for division of current between two parallel branches :
10W to 250 corresponds to iL =

i2

In the present context,

0:2 =

and

0:5 =

R1

R1

+ R2

IN R N
RN

+ 250

IN R N
RN

+ 80

(3.23)
(3.24)

Circuit Theorems

221

Solving equations (3.23) and (3.24), we get


IN
RN

= 1:7 A
= 33:33

(a) If vL iL is maximum,
= RN = 33:33
33:33
iL = 1:7
33:33 + 33:33
= 850 mA

RL

vL

(b) vL = IN (RN
.
RL =
Then, iL = 0 and

jj

= iL RL = 850

 10  33 33
3

= 28:33 V
RL )

is a maximum when

vL

jj

RN RL

is a maximum, which occurs when


= 1 7  33 33
= 1:7

RN

= 56:66 V
(c)

3.5

iL

IN R N
RN

+ RL

is maxmimum when RL = 0
iL

= 1:7A and vL = 0 V

Sinusoidal steady state analysis using superposition, Thevenin and


Norton equivalents

Circuits in the frequency domain with phasor currents and voltages and impedances are analogous
to resistive circuits.
To begin with, let us consider the principle of superposition, which may be restated as follows :
For a linear circuit containing two or more independent sources, any circuit voltage or
current may be calculated as the algebraic sum of all the individual currents or voltages caused
by each independent source acting alone.

Figure 3.101 Thevenin equivalent circuit

Figure 3.102

Norton equivalent circuit

222

Network Theory

The superposition principle is particularly useful if a circuit has two or more sources acting
at different frequencies. The circuit will have one set of impedance values at one frequency and a
different set of impedance values at another frequency. Phasor responses corresponding to different frequencies cannot be superposed; only their corresponding sinusoids can be superposed. That
is, when frequencies differ, the principle of superposition applies to the summing of time domain
components, not phasors. Within a component, problem corresponding to a single frequency,
however phasors may be superposed.
Thevenin and Norton equivalents in phasor circuits are found exactly in the same manner
as described earlier for resistive circuits, except for the subtitution of impedance Z in place of
resistance R and subsequent use of complex arithmetic. The Thevenin and Norton equivalent
circuits are shown in Fig. 3.101 and 3.102.
The Thevenin and Norton forms are equivalent if the relations
(a) Zt = ZN

(b) Vt = ZN IN

hold between the circuits.


A step by step procedure for finding the Thevenin equivalent circuit is as follows:
1. Identify a seperate circuit portion of a total circuit.
2. Find Vt = Voc at the terminals.
3.

(a) If the circuit contains only impedances and independent sources, then deactivate all the
independent sources and then find Zt by using circuit reduction techniques.
(b) If the circuit contains impedances, independent sources and dependent sources, then
either shortcircuit the terminals and determine Isc from which
Zt =

Voc
Isc

or deactivate the independent sources, connect a voltage or current source at the terminals, and
determine both V and I at the terminals from which
Zt =

V
I

A step by step procedure for finding Norton equivalent circuit is as follows:


(i) Identify a seperate circuit portion of the original circuit.
(ii) Short the terminals after seperating a portion of the original circuit and find the current
through the short circuit at the terminals, so that IN = Isc .
(iii)

(a) If the circuit contains only impedances and independent sources, then deactivate all the
independent sources and then find ZN = Zt by using circuit reduction techniques.
(b) If the circuit contains impedances, independent sources and one or more dependent
Voc
sources, find the opencircuit voltage at the terminals, Voc , so that ZN = Zt =
:
Isc

Circuit Theorems

EXAMPLE

223

3.37

Find the Thevenin and Norton equivalent circuits at the terminals


Fig. 3.103.

for the circuit in

Figure 3.103
SOLUTION

As a first step in the analysis, let us find Vt :

Using the principle of current division,


8 (4 /0 )
32
=
8 + j 10 j 5
8 + j5
j 320
Vt = Io (j 10) =
= 33:92 /58 V
8 + j5
Io =

To find Zt , deactivate all the independent sources. This results in a circuit diagram as shown
in Fig. 3.103 (a).

Figure 3.103(a)

Figure 3.103(b) Thevenin equivalent circuit

224

Network Theory

jj

Zt = j 10 (8 j 5)
(j 10)(8 j 5)
=
j 10 + 8
j5
= 10 /26
The Thevenin equivalent circuit as
viewed from the terminals a
b is
as shown in Fig 3.103(b). Performing
source transformation on the Thevenin
equivalent circuit, we get the Norton
equivalent circuit.

Figure : Norton equivalent circuit

Vt
33:92 /58
=
Zt
10 /26
= 3:392 /32 A

IN =

ZN = Zt = 10 /26
EXAMPLE

3.38

Find vo using Thevenins theorem. Refer to the circuit shown in Fig. 3.104.

Figure 3.104

SOLUTION

Let us convert the circuit given in Fig. 3.104 into a frequency domain equiavalent or phasor circuit
(shown in Fig. 3.105(a)). ! = 1
10 cos (t

45 )

5 sin (t + 30 ) = 5 cos (t

60 )

L
C

!
= 1F !

= 1H

j !L

1
j !C

!
!
=j
=

10 / 45 V
5 / 60 V

11= 1
1
11 = 1

Circuit Theorems

225

Figure 3.105(a)

Disconnecting the capicator from the original circuit, we get the circuit shown in
Fig. 3.105(b). This circuit is used for finding Vt .

Figure 3.105(b)

KCL at node a :
Vt

10 / 45
Vt 5 / 60
+
=0
3
j1
Vt = 4:97 / 40:54

Solving;

To find Zt deactivate all the independent sources


in Fig. 3.105(b). This results in a network as
shown in Fig. 3.105(c) :
Zt = Zab = 3 j 1
j3
3
=
=
(1 + j 3)
3+j
10

Figure 3.105(c)

jj

The Thevenin equivalent circuit along with the capicator


is as shown in Fig 3.105(d).
Vt
( j 1)
Zt j 1
4:97 / 40:54
(
=
0:3(1 + j 3) j 1
= 15:73 /247:9 V

Vo =

Hence;

vo

j 1)

= 15:73 cos (t + 247:9 ) V

Figure 3.105(d) Thevenin equivalent circuit

226

Network Theory

EXAMPLE

3.39

Find the Thevenin equivalent circuit of the circuit shown in Fig. 3.106.

Figure 3.106

SOLUTION

Since terminals a

are open,
Va = Is

 10

= 20 /0 V
Applying KVL clockwise for the mesh on the right hand side of the circuit, we get
3Va + 0 (j 10) + Voc

Va = 0

Voc = 4Va
= 80 /0 V
Let us transform the current source with 10 parallel resistance to a voltage source with 10
series resistance as shown in figure below :

To find Zt , the independent voltage source is deactivated and a current source of I A is


connected at the terminals as shown below :

Circuit Theorems

Applying KVL clockwise we get,


V0a

3V0a
0

4Va

j 10I

+ Vo = 0

j 10I

+ Vo = 0
V0a = 10I

Since

= Vo
Vo
= 40 + j 10
Zt =
I

we get

40I

Hence;

j 10I

Hence the Thevenin equivalent circuit is as shown


in Fig 3.106(a) :

EXAMPLE

Figure 3.106(a)

3.40

Find the Thevenin and Norton equivalent circuits for the circuit shown in Fig. 3.107.

Figure 3.107
SOLUTION

The phasor equivalent circuit of Fig. 3.107 is shown in Fig. 3.108.


KCL at node a :
Voc

2Voc
j 10

Voc =

Voc
=0
j5
100 100
j
=
/ 90 V
3
3
10 +

227

228

Network Theory

Figure 3.108

To find Isc , short the terminals a

of Fig. 3.108 as in Fig. 3.108(a).

Figure 3.108 (a)

Figure 3.108 (b)

Since Voc = 0, the above circuit takes the form shown in Fig 3.108 (b).
Isc = 10 /0 A
100
/ 90
Voc
10
3
Hence;
Zt =
=
=
/ 90

Isc
10 /0
3
The Thevenin equivalent and the Norton equivalent circuits are as shown below.

Figure Thevenin equivalent

EXAMPLE

Figure

Norton equivalent

3.41

Find the Thevenin and Norton equivalent circuits in frequency domain for the network shown in
Fig. 3.109.

Circuit Theorems

Figure 3.109

SOLUTION

Let us find Vt = Vab using superpostion theorem.


(i) Vab due to 100 /0

100 /0
100
=
A
j 300 + j 100
j 200
= I1 (j 100)
100
(j 100) = 50 /0 Volts
=
j 200

I1 =
Vab1

(ii) Vab due to 100 /90

229

230

Network Theory

100 /90
j 100
j 300
Vab2 = I2 ( j 300)
100 /90
=
(
j 100
j 300
Vt = Vab1 + Vab2
I2 =

Hence;

j 300)

= j 150 V

50 + j 150

= 158:11 /108:43 V
To find Zt , deactivate all the independent sources.

jj

Zt = j 100
j 300
j 100( j 300)
=
= j 150
j 100
j 300
Hence the Thevenin equivalent circuit is as shown in Fig. 3.109(a). Performing source transformation on the Thevenin equivalent circuit, we get the Norton equivalent circuit.
158:11 /108:43
Vt
=
= 1:054 /18:43 A
Zt
150 /90
ZN = Zt = j 150
IN =

The Norton equivalent circuit is as shown in Fig. 3.109(b).

Figure 3.109(a)

Figure 3.109(b)

Circuit Theorems

3.6

231

Maximum power transfer theorem

We have earlier shown that for a resistive network, maximum power is transferred from a source to
the load, when the load resistance is set equal to the Thevenin resistance with Thevenin equivalent
source. Now we extend this result to the ac circuits.

Figure 3.110 Linear circuit

Figure 3.111

Thevenin equivalent circuit

In Fig. 3.110, the linear circuit is made up of impedances, independent and dependent sources.
This linear circuit is replaced by its Thevenin equivalent circuit as shown in Fig. 3.111. The load
impedance could be a model of an antenna, a TV, and so forth. In rectangular form, the Thevenin
impedance Zt and the load impedance ZL are
Zt = Rt + jXt
ZL = RL + jXL

and
The current through the load is
I=

Vt
Vt
=
Zt + ZL
(Rt + jXt ) + (RL + jXL )

The phasors I and Vt are the maximum values. The corresponding RM S values are obtained
by dividing the maximum values by 2. Also, the RM S value of phasor current flowing in the
load must be taken for computing the average power delivered to the load. The average power
delivered to the load is given by
P

=
=

jj

1 2
I RL
2

jV j
t
2

2 RL
2

(Rt + RL ) (Xt + XL )2

(3.25)

Our idea is to adjust the load parameters RL and XL so that P is maximum. To do this, we
get

@P

@RL

and

@P

@XL

equal to zero.

232

Network Theory

@P
@XL

@P
@RL

j j
Vt

RL (Xt

+ XL )

(Rt + RL )2 + (Xt + XL )2

j j
Vt

i2

(Rt + RL )2 + (Xt + XL )2

2 (Rt + RL )2 + (Xt + XL )2
@P

Setting

@XL
XL
@P

and Setting

@RL
RL

2RL (Rt + RL )

i2

= 0 gives
=

(3.26)

Xt

= 0 gives

Rt2

+ (Xt + XL )2

(3.27)

Combining equations (3.26) and (3.27), we can conclude that for maximum average power
transfer, ZL must be selected such that XL = Xt and RL = Rt . That is the maximum average power of a circuit with an impedance Zt that is obtained when ZL is set equal to complex
conjugate of Zt .
Setting RL = Rt and XL = Xt in equation (3.25), we get the maximum average power as
P

j j
=
Vt

8Rt
In a situation where the load is purely real, the condition for maximum power transfer is
obtained by putting XL = 0 in equation (3.27). That is,
RL

Rt2

j j

+ Xt2 = Zt

Hence for maximum average power transfer to a purely resistive load, the load resistance is
equal to the magnitude of Thevenin impedance.
3.6.1 Maximum Power Transfer When Z is Restricted

Maximum average power can be delivered to ZL only if ZL = Zt . There are few situations in
which this is not possible. These situations are described below :
(i)

RL and XL may be restricted to a limited range of values. With this restriction,


choose
q XL as close as possible to Xt and then adjust RL as close as possible to
Rt2

+ (XL + Xt )2 :

(ii) Magnitude of ZL can be varied but its phase angle cannot be. Under this restriction,
greatest amount of power is transferred to the load when [ZL ] = Zt .

j j

Zt is the complex conjugate of Zt .

Circuit Theorems

EXAMPLE

233

3.42

Find the load impedance that transfers the maximum power to the load and determine the maximum power quantity obtained for the circuit shown in Fig. 3.112.

Figure 3.112
SOLUTION

We select, ZL = Zt for maximum power transfer.


ZL = 5 + j 6
10 /0
= 1 /0
I=
5+5

Hence

Hence, the maximum average power transfered to the


load is
P

EXAMPLE

jj

1 2
I RL
2
1
= (1)2 5 = 2:5 W
2
=

3.43

Find the load impedance that transfers the maximum average power to the load and determine the
maximum average power transferred to the load ZL shown in Fig. 3.113.

Figure 3.113

234

Network Theory

SOLUTION

The first step in the analysis is to find the Thevenin equivalent circuit by disconnecting the load
ZL . This leads to a circuit diagram as shown in Fig. 3.114.

Figure 3.114

Vt = Voc = 4 /0

Hence

3

= 12 /0 Volts(RMS)
To find Zt , let us deactivate all the independent sources of Fig. 3.114. This leads to a circuit
diagram as shown in Fig 3.114 (a):
Zt = 3 + j 4

Figure 3.114 (a)

Figure 3.115

The Thevenin equivalent circuit with ZL is as shown in Fig. 3.115.


For maximum average power transfer to the load, ZL = Zt = 3 j 4.
It =

12 /0
3 + j4 + 3

j4

= 2 /0 A(RMS)

Hence, maximum average power delivered to the load is


P

jj
It

RL

= 4(3) = 12 W

1
It may be noted that the scaling factor
is not taken since the phase current is already
2
expressed by its RM S value.

Circuit Theorems

EXAMPLE

235

3.44

Refer the circuit given in Fig. 3.116. Find the value of RL that will absorb the maximum average
power.

Figure 3.116

SOLUTION

Disconnecting the load resistor RL from the original circuit diagram leads to a circuit diagram as
shown in Fig. 3.117.

Figure 3.117

Vt = Voc = I1 (j 20)
150 /30
j 20
=
(40 j 30 + j 20)
= 72:76 /134 Volts:

To find Zt , let us deactivate all the independent sources present in Fig. 3.117 as shown in
Fig 3.117 (a).

jj

Zt = (40 j 30) j 20
j 20 (40
j 30)
=
= (9:412 + j 22:35)
j 20 + 40
j 30

236

Network Theory

The Value of RL that will absorb the maximum


average power is
RL

j j

= Zt =

(9:412)2 + (22:35)2

= 24:25
The Thevenin equivalent circuit with RL inserted
is as shown in Fig 3.117 (b).
Maximum average power absorbed by RL is
Pmax

where

1
2

jj
It

Figure 3.117 (a)

RL

72:76 /134
(9:412 + j 22:35 + 24:25)
= 1:8 /100:2 A
1
2
Pmax = (1:8)
24:25
2
= 39:29 W
It =

Figure 3.117 (b) Thevenin equivalent circuit

EXAMPLE

3.45

For the circuit of Fig. 3.118: (a) what is the value of ZL that will absorb the maximum average
power? (b) what is the value of maximum power?

Figure 3.118

SOLUTION

Disconnecting ZL from the original circuit we get the circuit as shown in Fig. 3.119. The first
step is to find Vt .

Circuit Theorems

Vt = Voc = I1 (

j 10)

120 /0
(
10 + j 15 j 10

j 10)

= 107:33 / 116:57 V
The next step is to find Zt . This requires deactivating the independent
voltage source of Fig. 3.119.

jj

Zt = (10 + j 15) (
=
=8

Figure 3.119

j 10)

j 10 (10
j 10

+ j 15)
+ 10 + j 15

j 14

The value of ZL for maximum average power absorbed is


Zt = 8 + j 14
The Thevenin equivalent circuit along with ZL = 8 + j 14 is as shown below:

107:33 / 116:57
8 j 14 + 8 + j 14
107:33
=
/ 116:57 A
16
1
2
Pmax =
It R L
2


1 107:33 2
8
=
2
16
= 180 Walts
It =

Hence;

jj

237

238

Network Theory

EXAMPLE

3.46

(a) For the circuit shown in Fig. 3.120, what is the value of ZL that results in maximum average
power that will be transferred to ZL ? What is the maximum power ?
(b) Assume that the load resistance can be varied between 0 and 4000 and the capacitive
reactance of the load can be varied between 0 and 2000 . What settings of RL and XC
transfer the most average power to the load ? What is the maximum average power that can
be transferred under these conditions?

Figure 3.120
SOLUTION

(a) If there are no constraints on RL and XL , the load indepedance ZL = Zt = (3000 j 4000) .
Since the voltage source is given in terms of its RM S value, the average maximum power
delivered to the load is
Pmax

RL

10 /0
3000 + j 4000 + 3000
10
=
A
2 3000
2
Pmax = It RL
100
=
3000
4 (3000)2
= 8:33 mW


jj


)
(b) Since
=

It =

where

XC

jj

= It

RL

and

XC

Thus,

are restricted, we firstqset

2000 . Next we set RL as close to

RL

j 4000

Rt2

XC

as close to

4000 as possible; hence

+ (XC + XL ) as possible.

30002 + ( 2000 + 4000)2 = 3605:55

Since RL can be varied between 0 to 4000 , we can set


adjusted to a value
ZL = 3605:55 j 2000 :

RL

to 3605:55 . Hence ZL is

Circuit Theorems

10 /0
3000 + j 4000 + 3605:55
= 1:4489 / 16:85 mA

It =

239

j 2000

The maximum average power delivered


to the load is
Pmax

jj

= It

RL

= 1:4489

 10

3 2

= 7:57 mW

 3605 55
:

Note that this is less than the power that can be delivered if there are no constraints on
and XL .
EXAMPLE

RL

3.47

A load impedance having a constant phase angle of 45 is connected across the load terminals
a and b in the circuit shown in Fig. 3.121. The magnitude of ZL is varied until the average power
delivered, which is the maximum possible under the given restriction.
(a) Specify ZL in rectangular form.
(b) Calculate the maximum average power delivered under this condition.

Figure 3.121

SOLUTION

Since the phase angle of ZL is fixed at


that

jZ j = pjZ j
t

(3000)2 + (4000)2

= 5000 :
Hence;

j j
p

ZL = ZL / 45
5000
5000
j
=
2
2

45 , for maximum power transfer to ZL it is mandatory

240

Network Theory

It =

10 /0
(3000 + 3535:53) + j (4000

3535:53)

= 1:526 / 4:07 mA
Pmax

jj

= It

RL

= 1:526

 10

3 2

 3535 53
:

= 8:23 mW
This power is the maximum average power that can be delivered by this circuit to a load
impedance whose angle is constant at 45 . Again this quantity is less than the maximum
power that could have been delivered if there is no restriction on ZL . In example 3.46 part (a),
we have shown that the maximum power that can be delivered without any restrictions on ZL
is 8.33 mW.

3.7

Reciprocity theorem

The reciprocity theorem states that in a linear bilateral single source circuit, the ratio of excitation to response is constant when the positions of excitation and response are interchanged.
Conditions to be met for the application of reciprocity theorem :

(i) The circuit must have a single source.


(ii) Initial conditions are assumed to be absent in the circuit.
(iii) Dependent sources are excluded even if they are linear.
(iv) When the positions of source and response are interchanged, their directions should be marked
same as in the original circuit.
EXAMPLE

3.48

Find the current in 2 resistor and hence verify reciprocity theorem.

Figure 3.122

Circuit Theorems

241

SOLUTION

The circuit is redrawn with markings as shown in Fig 3.123 (a).

Figure 3.123 (a)

Then;

+2

R1

= (8

R2

= 1:6 + 4 = 5:6

= 1:6

= (5:6 1 + 4 1 ) 1 = 2:3333
20
= 3:16 A
Current supplied by the source =
4 + 2:3333
4
= 1:32 A
Current in branch ab = Iab = 3:16
4 + 4 + 1: 6
8
= 1:05 A
Current in 2; I1 = 1:32
10
R3

Verification using reciprocity theorem


The circuit is redrawn by interchanging the position of excitation and response as shown in
Fig 3.123 (b).

Figure 3.123 (b)

Solving the equivalent resistances,


R4

= 2;

R5

= 6;

R6

= 3:4286

Now the current supplied by the source


=

20
= 3:6842A
3:4286 + 2

242

Network Theory

Therefore,
Icd

= 3:6842

 8 +8 6 = 2 1053A
:

I2

2:1053
= 1:05A
2

As I1 = I2 = 1:05 A, reciprocity theorem is verified.


EXAMPLE

3.49

In the circuit shown in Fig. 3.124, find the current through 1:375 resistor and hence verify
reciprocity theorem.

Figure 3.124
SOLUTION

Figure 3.125

KVL clockwise for mesh 1 :


6:375I1

2I2

3I3 = 0

KVL clockwise for mesh 2 :


2I1 + 14I2

10I3 = 0

KVL clockwise for mesh 3 :


3I1

10I2 + 14I3 =

10

Circuit Theorems

Putting the above three mesh equations in matrix form, we get

2
4

6:375
2
3

2
14
10

32
54

3
10
14

I1
I2

3 2
5=4

I3

0
0
10

243

3
5

Using Cramers rule, we get


I1

2A

Negative sign indicates that the assumed direction of current flow should have been the other way.
Verification using reciprocity theorem :
The circuit is redrawn by interchanging the positions of excitation and response. The new circuit
is shown in Fig. 3.126.

Figure 3.126

The mesh equations in matrix form for the circuit shown in Fig. 3.126 is

2
4

6:375
2
3

2 3
14 10
10 14

32
54

I1
0

I2

3 2
5=4

I3

10
0
0

3
5

Using Cramers rule, we get


0

I3

Since I1 = I30 =
EXAMPLE

2A

2 A, the reciprocity theorem is verified.

3.50

Find the current Ix in the j 2 impedance and hence verify reciprocity theorem.

Figure 3.127

244

Network Theory

SOLUTION

With reference to the Fig. 3.127, the current through j 2 impepance is found using series parallel
reduction techniques.
Total impedance of the circuit is

jj

ZT = (2 + j 3) + ( j 5) (3 + j 2)
( j 5)(3 + j 2)
= 2 + j3 +
j5 + 3 + j2
= 6:537 /19:36
The total current in the network is
36 /0
6:537 /19:36
= 5:507 / 19:36 A

IT =

Using the principle of current division, we find that


IT ( j 5)
j5 + 3 + j2
= 6:49 / 64:36 A

Ix =

Verification of reciprocity theorem :


The circuit is redrawn by changing the positions of excitation and response. This circuit is shown
in Fig. 3.128.
Total impedance of the circuit shown in
Fig. 3.128 is

jj

Z0T = (3 + j 2) + (2 + j 3) ( j 5)
(2 + j 3) ( j 5)
= (3 + j 2) +
2 + j3 j5
= 9:804 /19:36
The total current in the circuit is
I0T =

36 /0
0
ZT

= 3:672 / 19:36 A

Figure 3.128

Using the principle of current division,


I0T ( j 5)
= 6:49 / 64:36 A
j5 + 2 + j3
It is found that Ix = Iy , thus verifying the reciprocity theorem.
Iy =

EXAMPLE

3.51

Refer the circuit shown in Fig. 3.129. Find current through the ammeter, and hence verify reciprocity theorem.

Circuit Theorems

Figure 3.129

SOLUTION

To find the current through the ammeter :


By inspection the loop equations for the circuit in Fig. 3.130 can
be written in the matrix form as

2
4

16
1
10

1
26
20

10
20
30

32
54

I1
I2

3 2
5=4

I3

0
0
50

3
5

Using Cramers rule, we get


I1

= 4:6 A

I2

= 5:4 A

Hence current through the ammeter = I2

I1

= 5:4 4:6 = 0:8A.

Figure 3.130

Verification of reciprocity theorem:


The circuit is redrawn by interchanging the positions of
excitation and response as shown in Fig. 3.131.
By inspection the loop equations for the circuit can be
written in matrix form as

2
4

15
0
10

0
25
20

10
20
31

32
54

I1
0

I2
0

I3

3 2
5=4

50
50
0

3
5

Using Cramers rule we get


0

I3

= 0:8 A

Figure 3.131

245

246

Network Theory

Hence, current through the Ammeter = 0.8 A.


It is found from both the cases that the response is same. Hence the reciprocity theorem is
verified.
EXAMPLE

3.52

Find current through 5 ohm resistor shown in Fig. 3.132 and hence verify reciprocity theorem.

Figure 3.132
SOLUTION

By inspection, we can write

2
4

12
0
0 2 + j 10
2
2

2
2
9

32
54

I1
I2
I3

3 2
5=4

20
20
0

3
5

Using Cramers rule, we get


I3 = 0:5376 / 126:25 A
Hence, current through 5 ohm resistor = 0:5376 / 126:25 A
Verification of reciprocity theorem:
The original circuit is redrawn by interchanging the excitation and response as shown in Fig.
3.133.

Figure 3.133

Circuit Theorems

Putting the three equations in matrix form, we get

2
4

12
0
0 2 + j 10
2
2

2
2
9

32
5 64

I01
I02
I03

3 2
75 = 4

0
0
20

247

3
5

Using Cramers rule, we get


I01 = 0:3876 / 2:35 A
I02 = 0:456 / 78:9 A
I02

Hence;

I01 =

0:3179

j 0:4335

= 0:5376 / 126:25 A
The response in both cases remains the same. Thus verifying reciprocity theorem.

3.8

Millmans theorem

It is possible to combine number of voltage sources or current sources into a single equivalent voltage or current source using Millmans theorem. Hence, this theorem is quite useful in
calculating the total current supplied to the load in a generating station by a number of generators
connected in parallel across a busbar.
En
Millmans theorem states that if n number of generators having generated emfs E1 , E2 ;
and internal impedances Z1 ; Z2 ;
Zn are connected in parallel, then the emfs and impedances
can be combined to give a single equivalent emf of E with an internal impedance of equivalent
value Z.





E1 Y1 + E2 Y2 + : : : + En Yn
Y1 + Y2 + : : : + Yn
1
Z=
Y1 + Y2 + : : : + Yn

E=

where
and



where Y1 ; Y2
Yn are the admittances corresponding to the internal impedances Z1 ; Z2
and are given by
1
Z1
1
Y2 =
Z2
..
.
1
Yn =
Zn

 Z

Y1 =



Fig. 3.134 shows a number of generators having emfs E1 ; E2


En connected in parallel
Zn are the respective internal impedances of the
across the terminals x and y . Also, Z1 ; Z2
generators.



248

Network Theory

Figure 3.134

The Thevenin equivalent circuit of Fig. 3.134 using Millmans theorem is shown in Fig. 3.135.
The nodal equation at x gives



)
)

E1 E E2 E
En E
+
+
+
=0
Z1
Z2
Zn
 1 1

E E
En
1
1
2
=E
+
+
+
+
+
+
Z1
Z2
Zn
Zn
 Z1 1 Z2
+ En Yn = E
E1 Y1 + E2 Y2 +
Z






Figure 3.135

where Z = Equivalent internal impedance.

 + E Y ] = EY
E Y + E Y +  + E Y
E=
Y
Y = Y + Y +  + Y
1
1
Z=
=
Y
Y + Y +  + Y

[E1 Y1 + E2 Y2 +

or

where
and
EXAMPLE

3.53

Refer the circuit shown in Fig. 3.136. Find the current through 10 resistor using Millmans
theorem.

Figure 3.136

Circuit Theorems

249

SOLUTION

Using Millmans theorem, the circuit shown in Fig. 3.136 is replaced by its Thevenin equivalent
circuit across the terminals P Q as shown in Fig. 3.137.
E=

E1 Y1 + E2 Y2 E3 Y3
Y1 + Y2 + Y3
22

1

1

12
1
1
1
+
+
5 12 4
= 10:13 Volts
1
R=
Y1 + Y2 + Y3
1
=
0:2 + 0:083 + 0:25
= 1:88
=

+ 48

12

Hence;
EXAMPLE

1
4

Figure 3.137

IL

E
R

+ 10

= 0:853 A

3.54

Find the current through (10

j 3)

using Millmans theorem. Refer Fig. 3.138.

Figure 3.138

SOLUTION

The circuit shown in Fig. 3.138 is replaced by its Thevenin equivalent circuit as seen from the
terminals, A and B using Millmans theorem. Fig. 3.139 shows the Thevenin equivalent circuit
along with ZL = 10 j 3 :

250

Network Theory

Figure 3.139

E=

E1 Y1 + E2 Y2 E3 Y3
Y1 + Y2 + Y3

1

100 /0

+ 90 /45

1
10

+ 80 /30

1
20

1
1
1
+
+
5 10 20

= 88:49 /15:66 V
Z=R=

1
=
Y1 + Y2 + Y3

1
1
5

1
10

1
20

= 2:86

I=

88:49 /15:66
E
=
= 6:7 /28:79 A
Z + ZL
2:86 + 10 j 3

E=

E1 Y1 + E2 Y2 + E3 Y3 + E4 Y4
Y1 + Y2 + Y3 + Y4

Alternately,

100

5

+ 90 45 10 1 + 80 30
5 1 + 10 1 + 20 1 + (10 j 3)

 20

= 70 /12 V
Therefore;

70 /12
10 j 3

= 6:7 /28:8 A
EXAMPLE

3.55

Refer the circuit shown in Fig. 3.140. Use Millmans theorem to find the current through (5+j 5)
impedance.

Circuit Theorems

251

Figure 3.140
SOLUTION

The original circuit is redrawn after performing source transformation of 5 A in parallel with 4
resistor into an equivalent voltage source and is shown in Fig. 3.141.

Figure 3.141

Treating the branch 5 + j 5 as a branch with Es = 0V ,


E1 Y1 + E2 Y2 + E3 Y3 + E4 Y4
EP Q =
Y1 + Y2 + Y3 + Y4
4 2 1 + 8 3 1 + 20 4 1
= 1
2 + 3 1 + 4 1 + (5 j 5) 1
= 8:14 /4:83 V

Therefore current in (5 + j 5) is
I=

8:14 /4:83
= 1:15 / 40:2 A
5 + j5

Alternately
EP Q with (5 + j 5) open
E1 Y1 + E2 Y2 + E3 Y3
Y1 + Y2 + Y3
4 2 1 + 8 3 1 + 20
=
2 1+3 1+4 1
= 8:9231V

EP Q =

4

252

Network Theory

Equivalent resistance R = (2 1 + 3
Therefore current in (5 + j 5) is
I

EXAMPLE

+4

1) 1

= 0:9231

8:9231
= 1:15 / 40:2 A
0:9231 + 5 + j 5

3.56

Find the current through 2 resistor using Millmans theorem.


in Fig. 3.142.

Refer the circuit shown

Figure 3.142
SOLUTION

The Thevenin equivalent circuit using Millmans theorem for the given problem is as shown in
Fig. 3.142(a).
E=

where

E1 Y1 + E2 Y2
Y1 + Y2

10 /10
=

1
+ 25 /90
3 + j4
1
1
+
3 + j4 5

1
5

= 10:06 /97:12 V
1
1
Z=
=
1
1
Y1 + Y2
+
3 + j4 5
= 2:8 /26:56
Hence;

IL =

E
10:06 /97:12
=
Z + 2 2:8 /26:56 + 2
= 2:15 /81:63 A

Figure 3.142(a)

Circuit Theorems

Reinforcement problems
R.P

3.1

Find the current in 2 resistor connected between A and B by using superposition theorem.

Figure R.P. 3.1

SOLUTION

Fig. R.P. 3.1(a), shows the circuit with 2V-source acting alone (4V-source is shorted).
Resistance as viewed from 2V-source is 2 + R1 ,

where

Hence;
Then;

R1

3  2


+1
12

5
(1:2 + 1) 12
= 1:8592
=
14:2
2
Ia =
= 0:5182 A
2 + 1:8592
12
Ib = Ia
= 0:438 A
12 + 1 + 1:2
3
I1 = 0:438
= 0:2628 A
5

Figure R.P. 3.1(a)

With 4V-source acting alone, the circuit is as shown in Fig. R.P. 3.1(b).

Figure R.P.3.1(b)

253

254

Network Theory

The resistance as seen by 4V-source is 3 + R2 where


R2

 2  12


+1
2

14
2:7143 2
= 1:1551
=
4:7143
4
Hence;
Ib =
= 0:9635 A
3 + 1:1551
Ib 2:7143
= 0:555 A
Thus;
I2 =
4:7143
Finally, applying the principle of superposition,

we get,

IAB = I1 + I2
= 0:2628 + 0:555
= 0:818 A

R.P

3.2

For the network shown in Fig. R.P. 3.2, apply superposition theorem and find the current I.

Figure R.P. 3.2


SOLUTION

Open the 5A-current source and retain the voltage source. The resulting network is as shown in
Fig. R.P. 3.2(a).

Figure R.P. 3.2(a)

Circuit Theorems

255

The impedance as seen from the voltage source is


Z = (4

j 2)

(8 + j 10) (
8 + j8

j 2)

Ia =

= 6:01 / 45

j 20

= 3:328 /135 A
Z
Next, short the voltage source and retain the current source. The resulting network is as shown
in Fig. R.P. 3.2 (b).
Here, I3 = 5A. Applying KVL for mesh 1 and mesh 2, we
get
Hence;

8I1 + (I1
and

(I2

I1 ) (

I2 ) (

5) j 10 + (I1
j 2)

+ (I2

5) (

j 2)

j 2)

=0

+ 4I2 = 0

Simplifying, we get
(8 + j 8)I1 + j 2I2 = j 50
and

j 2I1

+ (4

j 4)I2

j 10

Solving, we get


8 + j 8
j2
Ib = I2 =
8 + j 8

j 50
j 10

j2
4 j4
= 2:897 / 23:96 A
j2

Figure R.P. 3.2(b)

Since, Ia and Ib are flowing in opposite directions, we


have
I = Ia Ib = 6:1121 /144:78 A
R.P

3.3

Apply superposition theorem and find the voltage across 1 resistor. Refer the circuit shown in
Fig. R.P. 3.3. Take v1 (t) = 5 cos (t + 10 ) and i2 (t) = 3 sin 2t A.

Figure R.P. 3.3

256

Network Theory

SOLUTION

To begin with let us assume v1 (t) alone is acting. Accordingly, short 10V - source and open i2 (t).
The resulting phasor network is shown in Fig. R.P. 3.3(a).
!

= 1rad=sec

! 5 /10
= 1H !
1
= 1F !
1
= H!
2
1
1
= F!
2


5 cos (t + 10 )
L1
C1
L2
C2

V
= j1

j !L1
j !C1

j !L2

j !C2

j1

=j

j2

va (t)

Let us next assume that


in Fig. R.P. 3.3(b).
!

i2 (t)

C2
L2

= 5 cos [t + 10 ]

alone is acting.

The resulting network is shown

= 2 rad=sec

! 3 /0
1
= 1F !
= 1H !
1
1
= F!
2
1
= H!
2

3 sin 2t

L1

Figure R.P. 3.3(a)

Va = 5 /10 V

C1

j !C1
2
j !L1 = j 2
j !C2

j !L2

j1

Figure R.P. 3.3(b)

= j1

Vb = 3 /0
vb (t)

 1 +1 51 5 = 2 5 /33 7
j :

j :

= 2:5 sin [2t + 33:7 ] A

Finally with 10V-source acting alone, the network is as shown in Fig. R.P. 3.3(c). Since
= 0, inductors are shorted and capacitors are opened.
Hence, Vc = 10 V
Applying principle of superposition, we
get.
!

v2 (t)

= va (t) = vb (t) + Vc
= 5 cos (t + 10 ) + 2:5 sin (2t + 33:7 ) + 10Volts
Figure R.P. 3.3(c)

Circuit Theorems

R.P

257

3.4

Calculate the current through the galvanometer for the Kelvin double bridge shown in Fig. R.P.
3.4. Use Thevenins theorem. Take the resistance of the galvanometer as 30 .

Figure R.P. 3.4

SOLUTION

With G being open, the resulting network is as shown in Fig. R.P. 3.4(a).

Figure 3.4(a)

VA
I2

10
 100 = 450
 100 = 209 V
5 =01
10
=
=
= 1 66
45  5
45 + 5
15+
50
= 05+
 10
= I1

IB

I2

Hence;

VB

I2

IB

= 2:5 V
Thus;

VAB

= Vt = VA

VB

20
9

2:5 =

5
Volts
18

: I2

258

Network Theory

To find Rt , short circuit the voltage source. The resulting network is as shown in Fig. R.P. 3.4(b).

Figure R.P. 3.4 (b)

Transforming the between B , E and F into an equivalent Y , we get


RB

35

 10 = 7

50

RE

35 5
= 3:5 ;
50

RF

 10 = 1
50

The reduced network after transformation is as shown in Fig. R.P. 3.4(c).

Figure R.P. 3.4(c)

Hence;

RAB

350 100 4:5 1:5


+
+7
450
6
= 85:903
= Rt =

The Thevenins equivalent circuit as seen from A


and B with 30 connected between A and B is
as shown in Fig. R.P. 3.4(d).
5
18
IG =
= 2:4mA
85:903 + 30
Negative sign implies that the current flows from
B to A.
R.P

Figure R.P. 3.4(d)

3.5

Find Is and R so that the networks N1 and N2 shown in Fig. R.P. 3.5 are equivalent.

Circuit Theorems

259

Figure R.P. 3.5


SOLUTION

Transforming the current source in N1 into an equivalent voltage source, we get N3 as shown in
Fig. R.P. 3.5(a).
V
I R = IS R
(3.28)
From N3 , we can write,
From N2 we can write,
Also from N2 ,

I
V

)
)
)

V
V

10Ia

3=

2Ia

3=
3=
I

10

=3

(3.29)

For equivalence of N1 and N2 , it is requirred that equations


(3.28) and (3.29) must be same. Comparing these equations, we
get
IR
R

R.P

= 0:2

and
and

IS R
IS

=3
=

3
= 15A
0:2

3.6

Obtain the Nortons equivalent of the network shown in Fig. R.P. 3.6.

Figure R.P. 3.6

Figure R.P. 3.5(a)

260

Network Theory

SOLUTION

Terminals a and b are shorted. This results in a network as shown in Fig. R.P. 3.6(a)

Figure R.P. 3.6(a)

The mesh equations are


9I1 + 0I2

6I3 = 30

(3.30)

(ii)

0I1 + 25I2 + 15I3 = 30

(3.31)

(iii)

6I1 + 15I2 + 23I3 = 4VX = 4 (10I2 )

(i)

6I1

25I2 + 23I3 = 0

Solving equations (3.30), (3.31) and (3.32), we get


IN

= Isc = I3 = 1:4706A

With terminals ab open, I3 = 0. The corresponding equations are

Hence;
Then;
Hence;

9I1 = 30
30
A
I1 =
9

25I2 = 50
30
and I2 =
A
25
30
VX = 10I2 = 10
= 12 V
25
6I1 4VX
Vt = Voc = 15I2
=

Thus;

Rt

and

50 V
Voc
Isc

50
=
1:4706

34

Hence, Nortons equivalent circuit is as shown in Fig. R.P. 3.6(b).

Figure R.P. 3.6(b)

(3.32)

Circuit Theorems

R.P

261

3.7

For the network shown in Fig. R.P. 3.7, find the Thevenins equivalent to show that

and

Vt

Zt

V1

2
3

(1 + a + b

ab)

Figure R.P. 3.7


SOLUTION

With xy open, I1 =
Hence,
Voc

aV1

V1

= Vt = aV1 + I1 + bI1
= aV1 +
=

V1

aV1

V1

[1 + a + b

+b

V

aV1

ab]

With xy shorted, the resulting network is


as shown in Fig. R.P. 3.7(a).

Figure R.P. 3.7(a)

Applying KVL equations, we get


(i)
(ii)

)
)

I1

+ (I1
2I1

(I2

I1 )

I2 )

= V1

aV1

I2

= V1

aV1

(3.33)

+ I2 = aV1 + bI1
(3.34)

(1 + b) I1 + 2I2 = aV1

Solving equations (3.33) and (3.34), we get


Isc

= I2 =

V1

(1 + a + b
3 b

ab)

262

Network Theory

Hence;

Zt

=
=

R.P

Voc
Isc

(1 + a + b
2 V1 (1 + a + b

V1

ab)
ab)

(3

b)

3.8

Use Nortons theorem to determine


are in ohms.

in the network shown in Fig. R.P. 3.8. Resistance Values

Figure R.P. 3.8

SOLUTION

Let IAE = x and IEF = y . Then by applying KCL at various junctions, the branch currents are
marked as shown in Fig. R.P. 3.8(a). Isc = 125 x = IAB on shorting A and B .
Applying KVL to the loop ABC F EA, we get
0:04x + 0:01y + 0:02 (y

20) + 0:03 (x

105) = 0

0:07x + 0:03y = 3:55

(3.35)

Applying KVL to the loop EDC EF , we get


(x

30) 0:03 + (x

55) 0:02

(y

20) 0:02
0:05x

0:01y = 0
0:08y = 1:6

(3.36)

Circuit Theorems

263

Figure R.P. 3.8(a)

Solving equations (3.35) and (3.36), we get


x

Hence;

Isc

= 46:76 A
= IN = 120

= 78:24 A
The circuit to calculate
opened.

Rt

is as shown in Fig. R.P. 3.8(b). All injected currents have been

Rt

= 0:03 + 0:04 +

0:03 0:05
0:08

= 0:08875

Figure R.P. 3.8(b)

Figure R.P. 3.8(c)

264

Network Theory

The Nortons equivalent network is as shown in Fig. R.P. 3.8(c).


I

= 78:24

0 08875
 0 08875
+ 0 04
:

= 53:9A
R.P

3.9

For the circuit shown in Fig. R.P. 3.9, find R such that the maximum power delivered to the load
is 3 mW.

Figure R.P. 3.9

SOLUTION

For a resistive network, the maximum power delivered to the load is


2

Pmax

Vt

4Rt

The network with RL removed is as shown in Fig.


R.P. 3.9(a).
Let the opent circuit voltage between the terminals a and b be Vt .
Then, applying KCL at node a, we get
1

Vt
R

Vt
R

Simplifying we get
Vt = 2 Volts
With all voltage sources shorted, the resistance,
found as follows:
1

Rt
Rt

=
=

1
R
R

1
R

Vt
R

Rt

Figure R.P. 3.9(a)

=0

as viewed from the terminals,

1
R

3
R

and b is

Circuit Theorems

22
3
=
= 3 103
R
R
4 3
R = 1 k

Hence,

Pmax =

R.P

| 265

3.10

Refer Fig. R.P. 3.10, find X1 and X2 interms of R1 and R2 to give maximum power dissipation
in R2 .

Figure R.P. 3.10


SOLUTION

The circuit for finding Zt is as shown in Figure R.P. 3.10(a).


Zt =
=

R1 (jX1 )
R1 + jX1

R1 X12 + jR12 X1
R12 + X12
Figure R.P. 3.10(a)

For maximum power transfer,


ZL = Zt

Hence,

R2 + jX2 =
R2 =

R1 X12
R12 X1

j
R12 + X12
R12 + X12
R1 X12
R12 + X12

X1 = R1
X2 =

R2
R1 R2

R12 X1
R12 + X12

Substituting equation (3.37) in equation (3.38) and simplifying, we get



X2 = R2 (R1 R2 )

(3.37)
(3.38)

266

Network Theory

Exercise Problems
E.P

3.1

Find ix for the circuit shown in Fig. E.P. 3.1 by using principle of superposition.

Ans : ix =
E.P

1
A
4

Figure E.P. 3.1

3.2

Find the current through branch P Q using superposition theorem.

Figure E.P. 3.2

Ans :
E.P

1.0625 A
3.3

Find the current through 15 ohm resistor using superposition theorem.

Figure E.P. 3.3

Ans :

0.3826 A

Circuit Theorems

E.P

3.4

Find the current through 3 + j 4 using superposition theorem.

Figure E.P. 3.4

Ans :
E.P

8.3 /85.3 A
3.5

Find the current through Ix using superposition theorem.

Figure E.P. 3.5

Ans :
E.P

3.07 / 163.12 A
3.6

Determine the current through 1 resistor using superposition theorem.

Figure E.P. 3.6

Ans :

0.406 A

267

268

E.P

Network Theory

3.7

Obtain the Thevenin equivalent circuit at terminals a

of the network shown in Fig. E.P. 3.7.

Figure E.P. 3.7

Ans :
E.P

Vt = 6.29 V, Rt = 9.43
3.8

Find the Thevenin equivalent circuit at terminals x

of the circuit shown in Fig. E.P. 3.8.

Figure E.P. 3.8

Ans :
E.P

Vt = 0.192 / 43.4 V, Zt = 88.7 /11.55


3.9

Find the Thevenin equivalent of the network shown in Fig. E.P. 3.9.

Figure E.P. 3.9

Ans :

Vt = 17.14 volts, Rt = 4

Circuit Theorems

E.P

3.10

Find the Thevenin equivalent circuit across a

b.

Refer Fig. E.P. 3.10.

Figure E.P. 3.10

Ans :
E.P

Vt =

30 V, Rt = 10 k

3.11

Find the Thevenin equivalent circuit across a

for the network shown in Fig. E.P. 3.11.

Figure E.P. 3.11

Ans :
E.P

Verify your result with other methods.


3.12

Find the current through 20 ohm resistor using Norton equivalent.

Figure E.P. 3.12

Ans :

IN = 4.36 A, RN = Rt = 8.8 , IL = 1.33 A

269

270

E.P

Network Theory

3.13

Find the current in 10 ohm resistor using Nortons theorem.

Figure E.P. 3.13

Ans :
E.P

IN =

4 A, Rt = RN

100
=
, IL =
7

0.5 A

3.14

Find the Norton equivalent circuit between the terminals a

b for the network shown in

Fig. E.P. 3.14.

Figure E.P. 3.14

Ans :
E.P

IN

= 4.98310 / 5.71 A, Zt = ZN = 3.6 /23.1

3.15

Determine the Norton equivalent circuit across the terminals


Fig. E.P. 3.15.

Figure E.P. 3.15

Ans :

IN = 5 A, RN = Rt = 6

for the network shown in

Circuit Theorems

E.P

271

3.16

Find the Norton equivalent of the network shown in Fig. E.P. 3.16.

Figure E.P. 3.16

Ans :
E.P

IN = 8.87 A, RN = Rt = 43.89
3.17

Determine the value of RL for maximum power transfer and also find the maximum power transferred.

Figure E.P. 3.17

Ans :
E.P

RL = 1.92 , Pmax = 4.67 W


3.18

Calculate the value of ZL for maximum power transfer and also calculate the maximum power.

Figure E.P. 3.18

Ans :

ZL = (7.97 + j2.16), Pmax = 0.36 W

272

E.P

Network Theory

3.19

Determine the value of RL for maximum power transfer and also calculate the value of maximum
power.

Figure E.P. 3.19

Ans :
E.P

RL = 5.44 , Pmax = 2.94 W


3.20

Determine the value of ZL for maximum power transfer. What is the value of maximum power?

Figure E.P. 3.20

Ans :
E.P

ZL = 4.23 + j1.15 , Pmax = 5.68 Watts


3.21

Obtain the Norton equivalent across x

y.

Figure E.P. 3.21

Ans :
E.P

IN = ISC = 7.35A, Rt = RN = 1.52


3.22

Find the Norton equivalent circuit at terminals a

of the network shown in Fig. E.P. 3.22.

Circuit Theorems

273

Figure E.P. 3.22

Ans :
E.P

IN = 1.05 /251.6 A, Zt = ZN = 10.6 /45


3.23

Find the Norton equivalent across the terminals X

of the network shown in Fig. E.P. 3.23.

Figure E.P. 3.23

Ans :
E.P

IN = 7A, Zt = 8.19 / 55


3.24

Determine the current through 10 ohm resistor using Nortons theorem.

Figure E.P. 3.24

Ans :

0.15A

274

E.P

Network Theory

3.25

Determine the current I using Nortons theorem.

Figure E.P. 3.25

Ans :
E.P

Verify your result with other methods.


3.26

Find Vx in the circuit shown in Fig. E.P. 3.26 and hence verify reciprocity theorem.

Figure E.P. 3.26

Ans :
E.P

Vx = 9.28 /21.81 V
3.27

Find Vx in the circuit shown in Fig. E.P. 3.27 and hence verify reciprocity theorem.

Figure E.P. 3.27

Ans :

Vx = 10.23 Volts

Circuit Theorems

E.P

3.28

Find the current ix in the bridge circuit and hence verify reciprocity theorem.

Figure E.P. 3.28

Ans :
E.P

ix = 0.031 A
3.29

Find the current through 4 ohm resistor using Millmans theorem.

Figure E.P. 3.29

Ans :
E.P

I = 2.05 A
3.30

Find the current through the impedance of (10 + j 10) using Millmans theorem.

Figure E.P. 3.30

Ans :

3.384 /12.6 A

275

276

E.P

Network Theory

3.31

Using Millmans theorem, find the current flowing through the impedance of (4 + j 3) .

Figure E.P. 3.31

Ans :

3.64 / 15.23 A

You might also like